Você está na página 1de 142

 

  

CLEVELAND 
NATIONAL BOARD REVIEW HANDOUTS 
CHIROPRACTIC 
PART III  
COLLEGE 
L  OS ANGELES  VERSION 1.0 

 
 
CCCLA NATIONAL BOARD REVIEW 
 
PART III – Version 1.0 
 
TABLE OF CONTENTS 
 
 
1. Extended Matching Case Questions   p 3‐92        of 142 
 
2. Compiled Fact Sheets        p 93‐123   of 142 
 
3. Miscellaneous Factoids           p 124‐142 of 142 
 
PATIENT INFORMATION

SEX: M AGE: 34

SUBJECTIVE FINDINGS:

CHIEF COMPLAINT: Right arm gets cold and hurts.

MECHANISM OF INJURY: Began insidiously 1 year ago

QUALITY OF PAIN: tingling, achy

SEVERITY OF PAIN: moderate

AGGRAVATING FACTORS: driving car; right lateral flexion of neck

ASSOCIATED SYMPTOMS: none

RELIEVING FACTORS: lying supine; left lateral flexion of neck

PAST MEDICAL HISTORY: non-contributory

FAMILY HISTORY: Mother had Hodgkin’s lymphoma

OBJECTIVE FINDINGS:

VITALS:
LEFT BP: 114/76 mmHg
RESP: 14 cpm
PULSE: 78 bpm
TEMP: 98.7ºF

RANGE OF MOTION:
Cervical ROM is WNL.

ORTHOPEDIC TESTING:
Cervical compression negative.

LABORATORY STUDIES:
N/A.

OTHER FINDINGS:
Right BP: 90/68 mmHg

Part III Handouts v.1.0 Page 3 of 142


Answer the following three (3) questions based on the patient
information provided on the previous page.

01 . Which orthopedic/neurologic tests do you expect to be positive? (CHOOSE 3)

A. Bracelet test E. Phalen

B. Adson test F. Costoclavicular test

C. Cervical distraction G. Hoffman sign

D. Wright test H. Babinski sign

02 . Which physical exam findings might you see? (CHOOSE 3)

A. Diminished pulses on right arm E. Hemoptysis

B. Edema of right arm F. Claw hand

C. Raynaud’s phenomenon G. Cough

D. Axillary lymphadenopathy H. Dyspnea

03 . Initial case management for this condition includes: (CHOOSE 3)

A. Referral to oncologist E. Run lab for CEA levels

B. Referral to cardiovascular specialist F. Cervical ROM exercises

C. Stretching of shortened muscles G. CMT

D. Cryotherapy H. Brugger’s postural exercise

Part III Handouts v.1.0 Page 4 of 142


Answer the following three (3) questions based on the patient
information provided on the previous page.

01 . Which orthopedic/neurologic tests do you expect to be positive? (CHOOSE 3)

A. Bracelet test E. Phalen

B. Adson test F. Costoclavicular test

C. Cervical distraction G. Hoffman sign

D. Wright test H. Babinski sign

02 . Which physical exam findings might you see? (CHOOSE 3)

A. Diminished pulses on right arm E. Hemoptysis

B. Edema of right arm F. Claw hand

C. Raynaud’s phenomenon G. Cough

D. Axillary lymphadenopathy H. Dyspnea

03 . Initial case management for this condition includes: (CHOOSE 3)

A. Referral to oncologist E. Run lab for CEA levels

B. Referral to cardiovascular specialist F. Cervical ROM exercises

C. Stretching of shortened muscles G. CMT

D. Cryotherapy H. Brugger’s postural exercise

Part III Handouts v.1.0 Page 5 of 142


PATIENT INFORMATION

SEX: F AGE: 45

SUBJECTIVE FINDINGS:

CHIEF COMPLAINT: Left shoulder pain and stiffness

MECHANISM OF INJURY: Insidious

QUALITY OF PAIN: Achy; sharp with movement

SEVERITY OF PAIN: 6/10

AGGRAVATING FACTORS: Movement; lying on either side

ASSOCIATED SYMPTOMS: Upper left arm pain

RELIEVING FACTORS: Moist heat

PAST MEDICAL HISTORY: fractured radius, age 7

FAMILY HISTORY: Diabetes; mother

OBJECTIVE FINDINGS:

VITALS: 45 y F
LEFT BP: 144/95 mmHg
RESP: 14 cpm
PULSE: 80 bpm
TEMP: 98.6 F

RANGE OF MOTION:
Left shoulder abduction 60 degrees; external
rotation 5 degrees; flexion 60 degrees

ORTHOPEDIC TESTING:
Empty can and Lift-off unable to perform

LABORATORY STUDIES:
HbA1c 8.1 (elevated)

OTHER FINDINGS:
Left deltoid atrophy

L AP shoulder

Part III Handouts v.1.0 Page 6 of 142


Answer the following three (3) questions based on the patient
information provided on the previous page.

01 . Select the three (3) best diagnoses for this patient. (CHOOSE 3)

A. Type 2 diabetes mellitus E. Rotator cuff syndrome

B. Disuse muscle atrophy F. Adhesive capsulitis

C. Iron deficiency anemia G. Shoulder dislocation

D. Clavicle fracture H. HADD

02 . Select the three (3) best case management strategies for this condition. (CHOOSE 3)

A. Cryotherapy E. Cold laser

B. Grade 5 mobilization of GH joint F. Iron supplements

C. Codman’s exercises G. Cardiovascular training

D. Wall-walking exercises H. Biceps curls

03 . Select the three (3) risk factors/causes for this shoulder condition. (CHOOSE 3)

A. Hypothyroidism E. Multiparity

B. Hypertension F. Hypercholesterolemia

C. Cervical myofascial pain syndrome G. Diabetes mellitus

D. Migraine headache H. Immobilization

Part III Handouts v.1.0 Page 7 of 142


Answer the following three (3) questions based on the patient
information provided on the previous page.

01 . Select the three (3) best diagnoses for this patient. (CHOOSE 3)

A. Type 2 diabetes mellitus E. Rotator cuff syndrome

B. Disuse muscle atrophy F. Adhesive capsulitis

C. Iron deficiency anemia G. Shoulder dislocation

D. Clavicle fracture H. HADD

02 . Select the three (3) best case management strategies for this condition. (CHOOSE 3)

A. Cryotherapy E. Cold laser

B. Grade 5 mobilization of GH joint F. Iron supplements

C. Codman’s exercises G. Cardiovascular training

D. Wall-walking exercises H. Biceps curls

03 . Select the three (3) risk factors/causes for this shoulder condition. (CHOOSE 3)

A. Hypothyroidism E. Multiparity

B. Hypertension F. Hypercholesterolemia

C. Cervical myofascial pain syndrome G. Diabetes mellitus

D. Migraine headache H. Immobilization

Part III Handouts v.1.0 Page 8 of 142


PATIENT INFORMATION

SEX: F AGE: 44

SUBJECTIVE FINDINGS:

CHIEF COMPLAINT: Pain over 3rd right metacarpal for 6 days.

MECHANISM OF INJURY: Pain occurred after striking back of hand on door knob.

QUALITY OF PAIN: Sharp ache.

SEVERITY OF PAIN: Mild-to-moderate.

AGGRAVATING FACTORS: Gripping objects, palpation of affected bone.

ASSOCIATED SYMPTOMS: Chronic joint pain/deformity, both hands. Ecchymosis over right hand.

RELIEVING FACTORS: None noted by patient.

PAST MEDICAL HISTORY: Patient has been a transient most of her adult life; little prior medical
contact.

FAMILY HISTORY: Mother is believed to also have had joint pains.

OBJECTIVE FINDINGS:

VITALS:
LEFT BP: 155/98 mmHg
RESP: 17 cpm
PULSE: 80 bpm
TEMP: 99.0ºF

RANGE OF MOTION:
Reduced in both hands
and wrists.

ORTHOPEDIC TESTING:
Bracelet test positive bilateral.

LABORATORY STUDIES:
FTA-ABS positive. R

OTHER FINDINGS:
Generalized, firm, discrete,
non-tender lymphadenopathy.

Part III Handouts v.1.0 Page 9 of 142


Answer the following three (3) questions based on the patient
information provided on the previous page.

01 . Which of the following diagnoses apply to this case? (CHOOSE 3)

A. Contusion of right hand E. Osteomalacia

B. Syphilis F. Rheumatoid arthritis

C. Sarcoidosis G. Ochronosis

D. Fracture of metacarpal H. Scleroderma

02 . Which of the following radiographic findings are seen? (CHOOSE 3)

A. Swan-neck deformity E. Juxta-articular erosions

B. Old gun shot wound F. Metacarpal fracture

C. Boutonniere deformity G. Chondrocalcinosis

D. Periarticular osteopenia H. Subchondral sclerosis

03 . What is appropriate initial management in this case? (CHOOSE 3)

A. Cold laser E. Fish oil supplementation

B. Ice water bath for hand F. Adjust thumbs

C. Referral to infectious disease specialist G. Splint hand

D. Referral to orthopedist for casting H. Myofascial release to dorsum of right hand

Part III Handouts v.1.0 Page 10 of 142


Answer the following three (3) questions based on the patient
information provided on the previous page.

01 . Which of the following diagnoses apply to this case? (CHOOSE 3)

A. Contusion of right hand E. Osteomalacia

B. Syphilis F. Rheumatoid arthritis

C. Sarcoidosis G. Ochronosis

D. Fracture of metacarpal H. Scleroderma

02 . Which of the following radiographic findings are seen? (CHOOSE 3)

A. Swan-neck deformity E. Juxta-articular erosions

B. Old gun shot wound F. Metacarpal fracture

C. Boutonniere deformity G. Chondrocalcinosis

D. Periarticular osteopenia H. Subchondral sclerosis

03 . What is appropriate initial management in this case? (CHOOSE 3)

A. Cold laser E. Fish oil supplementation

B. Ice water bath for hand F. Adjust thumbs

C. Referral to infectious disease specialist G. Splint hand

D. Referral to orthopedist for casting H. Myofascial release to dorsum of right hand

Part III Handouts v.1.0 Page 11 of 142


PATIENT INFORMATION

SEX: M AGE: 32

SUBJECTIVE FINDINGS:

CHIEF COMPLAINT: Right posterior leg pain to lower calf

MECHANISM OF INJURY: Started in 6th hour of 8 hours driving car

QUALITY OF PAIN: Sharp, shooting

SEVERITY OF PAIN: 6/10

AGGRAVATING FACTORS: Crossing right leg over left; walking

ASSOCIATED SYMPTOMS: Ache in right buttock, low back tightness

RELIEVING FACTORS: Lying supine

PAST MEDICAL HISTORY: Unremarkable

FAMILY HISTORY: Unremarkable

OBJECTIVE FINDINGS:

VITALS:
LEFT BP: 112/74 mmHg
RESP: 14 cpm
PULSE: 78 bpm
TEMP: 97.7 F

RANGE OF MOTION:
Decreased internal rotation of right FAJ;
produces pain in leg.

ORTHOPEDIC TESTING:
Hibb test positive. Braggard positive.
Valsalva negative.

LABORATORY STUDIES:
ESR 2 mmHr (normal).

OTHER FINDINGS:
Extension restriction of right SI joint.
Scalp has reddened skin with greasy scales.

Part III Handouts v.1.0 Page 12 of 142


Answer the following three (3) questions based on the patient
information provided on the previous page.
KEY

01 . Select the three appropriate diagnoses. (CHOOSE 3)

A. Lumbar disc herniation E. Psoriatic arthritis

B. Sciatic radiculitis F. Piriformis syndrome

C. Sacroiliac dysfunction G. Hip OA

D. Ankylosing spondylitis H. Hamstring strain

02 . Which of the following are appropriate outcome measure to use? (CHOOSE 3)

A. VAS E. Zung

B. Rheumatoid disability index F. IPSS

C. Oswestry G. Fiese disability index

D. Lower extremity functional scale H. COOP

03 . Select the appropriate initial management for this patient. (CHOOSE 3)

A. Strengthening of right hip external rotators E. Cat/camel exercises

B. Referral to rheumatologist F. Manipulation of right SI joint

C. Cox flexion-distraction G. Bed rest

D. Ultrasound to right buttock H. PIR to right piriformis

Part III Handouts v.1.0 Page 13 of 142


Answer the following three (3) questions based on the patient
information provided on the previous page.

01 . Select the three appropriate diagnoses. (CHOOSE 3)

A. Lumbar disc herniation E. Psoriatic arthritis

B. Sciatic radiculitis F. Piriformis syndrome

C. Sacroiliac dysfunction G. Hip OA

D. Ankylosing spondylitis H. Hamstring strain

02 . Which of the following are appropriate outcome measure to use? (CHOOSE 3)

A. VAS E. Zung

B. Rheumatoid disability index F. IPSS

C. Oswestry G. Fiese disability index

D. Lower extremity functional scale H. COOP

03 . Select the appropriate initial management for this patient. (CHOOSE 3)

A. Strengthening of right hip external rotators E. Cat/camel exercises

B. Referral to rheumatologist F. Manipulation of right SI joint

C. Cox flexion-distraction G. Bed rest

D. Ultrasound to right buttock H. PIR to right piriformis

Part III Handouts v.1.0 Page 14 of 142


PATIENT INFORMATION

SEX: Male AGE: 65

SUBJECTIVE FINDINGS:

CHIEF COMPLAINT: Neck pain

MECHANISM OF INJURY: unknown, insidious onset

QUALITY OF PAIN: achy, stiff

SEVERITY OF PAIN: mild to moderate

AGGRAVATING FACTORS: worse after periods of inactivity

ASSOCIATED SYMPTOMS: excessive stumbling and tripping

RELIEVING FACTORS: Advil

PAST MEDICAL HISTORY: unremarkable

FAMILY HISTORY: Father had multiple sclerosis

OBJECTIVE FINDINGS:

VITALS:
LEFT BP: 132/88 mmHg
RESP: 16 cpm
PULSE: 92 bpm
TEMP: 98.3 F

RANGE OF MOTION:
Decreased cervical lateral flexion and rotation
bilaterally.

ORTHOPEDIC TESTING:
Increased neck pain with cervical compression.
Reduced neck pain with cervical distraction.

LABORATORY STUDIES:
CBC and Chem-7 unremarkable.

OTHER FINDINGS:
Loss of vibration and proprioception to the ankles
bilaterally.

Part III Handouts v.1.0 Page 15 of 142


Answer the following three (3) questions based on the patient
information provided on the previous page.

1. Select the three (3) best differential diagnoses. (CHOOSE 3)

A. Diabetes mellitus E. Degenerative cervical spine stenosis

B. Multiple sclerosis F. Friedreich’s ataxia

C. DISH G. Paget’s disease of cervical spine

D. Myelopathy H. Cauda equina syndrome

2. What additional studies should be performed? (CHOOSE 3)

A. MRI of spine E. Serum alkaline phosphatase

B. EEG F. Sensory evoked potentials

C. Carotid angiography G. Femoral angiography

D. Serum HLA-B27 H. EMG

3. Which of the following are the most appropriate forms of management? (CHOOSE 3)

A. Calcium supplements E. Grade 3 mobilization of neck

B. Cervical ultrasound F. Cryotherapy

C. Cervical traction G. Referral to neurosurgeon

D. Cervical collar H. Cervical extension exercises

Part III Handouts v.1.0 Page 16 of 142


Answer the following three (3) questions based on the patient
information provided on the previous page.

1. Select the three (3) best differential diagnoses. (CHOOSE 3)

A. Diabetes mellitus E. Degenerative cervical spine stenosis

B. Multiple sclerosis F. Friedreich’s ataxia

C. DISH G. Paget’s disease of cervical spine

D. Myelopathy H. Cauda equina syndrome

2. What additional studies should be performed? (CHOOSE 3)

A. MRI of spine E. Serum alkaline phosphatase

B. EEG F. Sensory evoked potentials

C. Carotid angiography G. Femoral angiography

D. Serum HLA-B27 H. EMG

3. Which of the following are the most appropriate forms of management? (CHOOSE 3)

A. Calcium supplements E. Grade 3 mobilization of neck

B. Cervical ultrasound F. Cryotherapy

C. Cervical traction G. Referral to neurosurgeon

D. Cervical collar H. Cervical extension exercises

Part III Handouts v.1.0 Page 17 of 142


PATIENT INFORMATION

SEX: F AGE: 37

SUBJECTIVE FINDINGS:

CHIEF COMPLAINT: Right medial forearm pain down to 3rd, 4th and 5th digits.

MECHANISM OF INJURY: Insidious onset.

QUALITY OF PAIN: Pulling, tingling, numbness of affected digits.

SEVERITY OF PAIN: Moderate.

AGGRAVATING FACTORS: Driving, reading newspaper.

ASSOCIATED SYMPTOMS: Fingers on the right periodically become cold, pale, then cyanotic.

RELIEVING FACTORS: Neck and shoulder massage.

PAST MEDICAL HISTORY: Cholecystectomy, three years ago.

FAMILY HISTORY: Unremarkable.

OBJECTIVE FINDINGS:

VITALS:
LEFT BP: 116/70 mmHg
RESP: 18 cpm
PULSE: 78 bpm
TEMP: 98.6 ºF

RANGE OF MOTION:
Reduced left lateral flexion and left
rotation of neck.

ORTHOPEDIC TESTING:
Jackson test negative bilaterally.

LABORATORY STUDIES:
None.

OTHER FINDINGS:
Upper thoracic hyperkyphosis with
intersegmental dysfunction at T3-T5.

Part III Handouts v.1.0 Page 18 of 142


Answer the following three (3) questions based on the patient
information provided on the previous page.

01 . What additional orthopedic tests are needed in this case? (CHOOSE 3)

A. Soto-Hall test E. Wright test

B. Allen test F. Schepelmann sign

C. Adson test G. Hawkin test

D. Valsalva maneuver H. Naffziger test

02 . Select the three most appropriate differential diagnoses. (CHOOSE 3)

A. Cervical discopathy with radiculopathy E. Pott’s disease

B. T4 syndrome F. Barrel chest

C. Raynaud’s phenomenon G. Cervical degenerative stenosis

D. Thoracic outlet syndrome H. Pancoast tumor

03 . Select the most appropriate initial management for this case. (CHOOSE 3)

A. McKenzie chin retractions E. Advise patient to stop smoking

B. Strengthen cervical erector spinae F. CMT to T3-T5

C. Ultrasound forearm G. Vitamin B12 supplements

D. CMT to midcervical spine H. Stretch scalenes and pecs

Part III Handouts v.1.0 Page 19 of 142


Answer the following three (3) questions based on the patient
information provided on the previous page.

01 . What additional orthopedic tests are needed in this case? (CHOOSE 3)

A. Soto-Hall test E. Wright test

B. Allen test F. Schepelmann sign

C. Adson test G. Hawkin test

D. Valsalva maneuver H. Naffziger test

02 . Select the three most appropriate differential diagnoses. (CHOOSE 3)

A. Cervical discopathy with radiculopathy E. Pott’s disease

B. T4 syndrome F. Barrel chest

C. Raynaud’s phenomenon G. Cervical degenerative stenosis

D. Thoracic outlet syndrome H. Pancoast tumor

03 . Select the most appropriate initial management for this case. (CHOOSE 3)

A. McKenzie chin retractions E. Advise patient to stop smoking

B. Strengthen cervical erector spinae F. CMT to T3-T5

C. Ultrasound forearm G. Vitamin B12 supplements

D. CMT to midcervical spine H. Stretch scalenes and pecs

Part III Handouts v.1.0 Page 20 of 142


PATIENT INFORMATION

SEX: F AGE: 36

SUBJECTIVE FINDINGS:

CHIEF COMPLAINT: Low back pain

MECHANISM OF INJURY: Moving boxes

QUALITY OF PAIN: Sharp, achy

SEVERITY OF PAIN: Severe

AGGRAVATING FACTORS: Sitting, coughing

ASSOCIATED SYMPTOMS: Sharp, shooting right leg pain to lateral foot

RELIEVING FACTORS: Lying down with knees bent

PAST MEDICAL HISTORY: Unremarkable

FAMILY HISTORY: RA, mother and sister

OBJECTIVE FINDINGS:

VITALS:
LEFT BP: 118/82 mmHg
RESP: 20 cpm
PULSE: 92 bpm
TEMP: 98.7 F

RANGE OF MOTION:
Lumbar flexion 15 degrees, extension
5 degrees.

ORTHOPEDIC TESTING:
Positive SLR, Braggard, Valsalva.

LABORATORY STUDIES:
None.

OTHER FINDINGS:
Achilles reflex +1.

Part III Handouts v.1.0 Page 21 of 142


Answer the following three (3) questions based on the patient
information provided on the previous page.

01 . Which three (3) additional ortho/neuro tests are expected to be positive? (CHOOSE 3)

A. L4 dermatome E. Hibb test

B. L5 dermatome F. Prone knee bend

C. Peronei muscle test G. Tibialis anterior muscle test

D. Kemp test H. Patellar reflex

02 . Select three (3) initial case management strategies. (CHOOSE 3)

A. McKenzie technique E. Cryotherapy

B. Hot pack F. Referral to orthopedic surgeon

C. 2 weeks bed rest G. Ultrasound (continuous mode)

D. SOT blocking H. Russian stim

03. Which three (3) outcome measures are most appropriate in this case? (CHOOSE 3)

A. Zung E. Duke health profile

B. Braden scale F. Visual analog scale

C. Rowland-Morris G. SF-36

D. COOP H. Oswestry

Part III Handouts v.1.0 Page 22 of 142


Answer the following three (3) questions based on the patient
information provided on the previous page.

01 . Which three (3) additional ortho/neuro tests are expected to be positive? (CHOOSE 3)

A. L4 dermatome E. Hibb test

B. L5 dermatome F. Prone knee bend

C. Peronei muscle test G. Tibialis anterior muscle test

D. Kemp test H. Patellar reflex

02 . Select three (3) initial case management strategies. (CHOOSE 3)

A. McKenzie technique E. Cryotherapy

B. Hot pack F. Referral to orthopedic surgeon

C. 2 weeks bed rest G. Ultrasound (continuous mode)

D. SOT blocking H. Russian stim

03. Which three (3) outcome measures are most appropriate in this case? (CHOOSE 3)

A. Zung E. Duke health profile

B. Braden scale F. Visual analog scale

C. Rowland-Morris G. SF-36

D. COOP H. Oswestry

Part III Handouts v.1.0 Page 23 of 142


PATIENT INFORMATION

SEX: F AGE: 22

SUBJECTIVE FINDINGS:

CHIEF COMPLAINT: Progressively severe unilateral wrist pain of 24 hr duration.

MECHANISM OF INJURY: While camping with new boyfriend; denies any trauma.

QUALITY OF PAIN: Deep ache.

SEVERITY OF PAIN: Severe; interrupts sleep.

AGGRAVATING FACTORS: Wrist movement.

ASSOCIATED SYMPTOMS: Progressive malaise and anorexia.

RELIEVING FACTORS: None.

PAST MEDICAL HISTORY: Noncontributory.

FAMILY HISTORY: Noncontributory.

OBJECTIVE FINDINGS:

VITALS:
LEFT BP: 118/76 mmHg
RESP: 18 cpm
PULSE: 98 bpm
TEMP: 101 ºF

RANGE OF MOTION:
Involved wrist ROM severely restricted.

ORTHOPEDIC TESTING:
Bracelet test produces excruciating wrist pain.

LABORATORY STUDIES:
WBC count 11500.

OTHER FINDINGS:
Bilateral pustular eruptions on hands.
Mild pharyngeal injection.

Part III Handouts v.1.0 Page 24 of 142


Answer the following three (3) questions based on the patient
information provided on the previous page.

01 . Which of the following are the most likely differential diagnoses? (CHOOSE 3)

A. Lyme disease, first stage E. Acute rheumatic fever

B. Disseminated gonococcal infection F. Wrist sprain

C. Streptococcal septic arthritis G. Wrist fracture

D. Reactive arthritis H. Psoriatic arthritis

02 . What additional clinical findings are expected in this case? (CHOOSE 3)

A. Purulent cervical discharge E. Cardiac murmur

B. Dysuria F. Subcutaneous nodules

C. Erythema migrans G. Bell’s palsy

D. Erythema marginatum H. Pelvic pain

03 . What additional exams, tests or imaging is indicated? (CHOOSE 3)

A. Echocardiogram E. Neurological exam

B. Chest x-ray F. Blood culture

C. Liver function tests G. Wrist xray

D. Pelvic exam H. HLA-B27

Part III Handouts v.1.0 Page 25 of 142


Answer the following three (3) questions based on the patient
information provided on the previous page.

01 . Which of the following are the most likely differential diagnoses? (CHOOSE 3)

A. Lyme disease, first stage E. Acute rheumatic fever

B. Disseminated gonococcal infection F. Wrist sprain

C. Streptococcal septic arthritis G. Wrist fracture

D. Reactive arthritis H. Psoriatic arthritis

02 . What additional clinical findings are expected in this case? (CHOOSE 3)

A. Purulent cervical discharge E. Cardiac murmur

B. Dysuria F. Subcutaneous nodules

C. Erythema migrans G. Bell’s palsy

D. Erythema marginatum H. Pelvic pain

03 . What additional exams, tests or imaging is indicated? (CHOOSE 3)

A. Echocardiogram E. Neurological exam

B. Chest x-ray F. Blood culture

C. Liver function tests G. Wrist xray

D. Pelvic exam H. HLA-B27

Part III Handouts v.1.0 Page 26 of 142


PATIENT INFORMATION

SEX: F AGE: 56

SUBJECTIVE FINDINGS:

CHIEF COMPLAINT: Polyarticular joint pain and swelling in the hands bilaterally.

MECHANISM OF INJURY: Insidious, 36 years ago.

QUALITY OF PAIN: Aching and prolonged morning stiffness.

SEVERITY OF PAIN: 3-4/10.

AGGRAVATING FACTORS: Typing, opening jars.

ASSOCIATED SYMPTOMS: Recurrent oral ulcers and intermittent rash. Recurrent neck pain.

RELIEVING FACTORS: NSAIDs.

PAST MEDICAL HISTORY: Pleuritis 8 years ago, pericarditis 12 years ago.

FAMILY HISTORY: Mother had some type of arthritis.

OBJECTIVE FINDINGS:

VITALS:
LEFT BP: 126/84 mmHg
RESP: 16 cpm
PULSE: 82 bpm
TEMP: 98.9 ºF

RANGE OF MOTION:
Finger flexion and extension
reduced bilaterally.

ORTHOPEDIC TESTING:
Bracelet test negative bilaterally.

LABORATORY STUDIES:
CBC: Normocytic anemia.

OTHER FINDINGS:
Raynaud’s phenomenon. (Exam
room is cold.)

Part III Handouts v.1.0 Page 27 of 142


Answer the following three (3) questions based on the patient
information provided on the previous page.

01 . Which of the following cause ulnar deviation of the digits? (CHOOSE 3)

A. Osteoarthritis E. Gout

B. Scleroderma F. Reactive arthritis

C. Rheumatoid arthritis G. Ollier’s disease

D. Psoriatic arthritis H. Systemic lupus erythematosis

02 . Which of the following additional lab tests or imaging is needed. (CHOOSE 3)

A. Flexion-extension cervical films E. Serum electrolytes

B. Serum anti-nuclear antibody F. Bone scan

C. Serum uric acid G. Serum anti-dsDNA antibody

D. HLA-B27 H. Chest MRI

03 . What additional clinical findings are expected in this case? (CHOOSE 3)

A. Lymphadenopathy E. Rheumatoid nodules

B. Endometriosis F. Glomerulonephritis

C. Photosensitivity G. Irritable bowel syndrome

D. Recurrent urinary tract infection H. Lipomas

Part III Handouts v.1.0 Page 28 of 142


Answer the following three (3) questions based on the patient
information provided on the previous page.

01 . Which of the following cause ulnar deviation of the digits? (CHOOSE 3)

A. Osteoarthritis E. Gout

B. Scleroderma F. Reactive arthritis

C. Rheumatoid arthritis G. Ollier’s disease

D. Psoriatic arthritis H. Systemic lupus erythematosis

02 . Which of the following additional lab tests or imaging is needed. (CHOOSE 3)

A. Flexion-extension cervical films E. Serum electrolytes

B. Serum anti-nuclear antibody F. Bone scan

C. Serum uric acid G. Serum anti-dsDNA antibody

D. HLA-B27 H. Chest MRI

03 . What additional clinical findings are expected in this case? (CHOOSE 3)

A. Lymphadenopathy E. Rheumatoid nodules

B. Endometriosis F. Glomerulonephritis

C. Photosensitivity G. Irritable bowel syndrome

D. Recurrent urinary tract infection H. Lipomas

Part III Handouts v.1.0 Page 29 of 142


PATIENT INFORMATION

SEX: M AGE: 28

SUBJECTIVE FINDINGS:

CHIEF COMPLAINT: Right groin pain when consuming alcohol.

MECHANISM OF INJURY: Insidious 2 months ago.

QUALITY OF PAIN: Achy.

SEVERITY OF PAIN: Mild.

AGGRAVATING FACTORS: Consumption of alcohol.

ASSOCIATED SYMPTOMS: Night sweats, sporadic high fever.

RELIEVING FACTORS: None.

PAST MEDICAL HISTORY: Treatment with phenytoin for grand mal seizures.

FAMILY HISTORY: SLE, mother.

OBJECTIVE FINDINGS:

VITALS:
LEFT BP: 124/78 mmHg
RESP: 16 cpm
PULSE: 78 bpm
TEMP: 97.5 F

RANGE OF MOTION:
Right hip ROM within normal limits.

ORTHOPEDIC TESTING:
Patrick FABERE negative.

LABORATORY STUDIES:
Lymphocytopenia.

OTHER FINDINGS:
Left lower leg edema.

Part III Handouts v.1.0 Page 30 of 142


Answer the following three (3) questions based on the patient
information provided on the previous page.

01 . Select three (3) additional lab test results that are expected in this case. (CHOOSE 3)

A. Thrombocytopenia E. Langerhans cells

B. Microcytic anemia F. Starry sky macrophages

C. Low TSH G. Hypercholesterolemia

D. Elevated ESR H. Reed-Sternberg cells

02 . Which of the following are possible consequences of treatment? (CHOOSE 3)

A. Breast cancer E. Multiple sclerosis

B. Stroke F. Lung cancer

C. Thyroid cancer G. HIV infection

D. Degenerative arthritis H. Chronic pancreatitis

03 . Select “red flag” findings for additional or follow-up assessment. (CHOOSE 3)

A. Hemoptysis E. Chest pain

B. Anxiety F. Sinusitis

C. Aphthous ulcers G. Constipation

D. Fordyce spots H. Mitral valve prolapse

Part III Handouts v.1.0 Page 31 of 142


Answer the following three (3) questions based on the patient
information provided on the previous page.

01 . Select three (3) additional lab test results that are expected in this case. (CHOOSE 3)

A. Thrombocytopenia E. Langerhans cells

B. Microcytic anemia F. Starry sky macrophages

C. Low TSH G. Hypercholesterolemia

D. Elevated ESR H. Reed-Sternberg cells

02 . Which of the following are possible consequences of treatment? (CHOOSE 3)

A. Breast cancer E. Multiple sclerosis

B. Stroke F. Lung cancer

C. Thyroid cancer G. HIV infection

D. Degenerative arthritis H. Chronic pancreatitis

03 . Select “red flag” findings for additional or follow-up assessment. (CHOOSE 3)

A. Hemoptysis E. Chest pain

B. Anxiety F. Sinusitis

C. Aphthous ulcers G. Constipation

D. Fordyce spots H. Mitral valve prolapse

Part III Handouts v.1.0 Page 32 of 142


PATIENT INFORMATION

SEX: F AGE: 47

SUBJECTIVE FINDINGS:

CHIEF COMPLAINT: Right second carpal digit DIP joint pain and swelling

MECHANISM OF INJURY: Insidious

QUALITY OF PAIN: Sharp ache

SEVERITY OF PAIN: Moderate

AGGRAVATING FACTORS: Prolonged inactivity, excessive activity

ASSOCIATED SYMPTOMS: Morning stiffness lasting 2 hours

RELIEVING FACTORS: NSAIDs

PAST MEDICAL HISTORY: Recurrent UTIs

FAMILY HISTORY: Non-contributory

OBJECTIVE FINDINGS:

VITALS:
LEFT BP: 120/80 mmHg
RESP: 14 cpm
PULSE: 74 bpm
TEMP: 98.1 F

RANGE OF MOTION:
Mildly reduced at affected
joint.

ORTHOPEDIC TESTING:
None.

LABORATORY STUDIES:
None.

OTHER FINDINGS:
Rash.

Part III Handouts v.1.0 Page 33 of 142


Answer the following three (3) questions based on the patient
information provided on the previous page.

01 . Which other anatomical sites can be affected in this condition? (CHOOSE 3)

A. DIP joints of toes E. Low back

B. Heart F. Pericardium

C. Nails G. Spleen

D. Eyes H. Oral mucosa

02 . Which three (3) radiology findings would you expect in this condition? (CHOOSE 3)

A. Symmetrical arthritis of carpal DIP joints E. Sudek atrophy

B. Resorption of terminal phalanges F. Asymmetric syndesmophytes

C. Hypertrophic osteophytes G. Subperiosteal new bone growth

D. Rat bite appearance H. Pencil in cup deformity

03 . Select the appropriate initial management of this patient. (CHOOSE 3)

A. Grade 5 mobilization of affected DIP joint E. Cross friction massage

B. Paraffin wax F. Hard cast

C. Hot pack G. Fish oil supplements

D. Referral to rheumatologist H. Subaqeous ultrasound (pulsed)

Part III Handouts v.1.0 Page 34 of 142


Answer the following three (3) questions based on the patient
information provided on the previous page.

01 . Which other anatomical sites can be affected in this condition? (CHOOSE 3)

A. DIP joints of toes E. Low back

B. Heart F. Pericardium

C. Nails G. Spleen

D. Eyes H. Oral mucosa

02 . Which three (3) radiology findings would you expect in this condition? (CHOOSE 3)

A. Symmetrical arthritis of carpal DIP joints E. Sudek atrophy

B. Resorption of terminal phalanges F. Asymmetric syndesmophytes

C. Hypertrophic osteophytes G. Subperiosteal new bone growth

D. Rat bite appearance H. Pencil in cup deformity

03 . Select the appropriate initial management of this patient. (CHOOSE 3)

A. Grade 5 mobilization of affected DIP joint E. Cross friction massage

B. Paraffin wax F. Hard cast

C. Hot pack G. Fish oil supplements

D. Referral to rheumatologist H. Subaqeous ultrasound (pulsed)

Part III Handouts v.1.0 Page 35 of 142


PATIENT INFORMATION

SEX: M AGE: 78

SUBJECTIVE FINDINGS:

CHIEF COMPLAINT: Low back pain

MECHANISM OF INJURY: Insidious

QUALITY OF PAIN: Deep, boring

SEVERITY OF PAIN: 9/10

AGGRAVATING FACTORS: None

ASSOCIATED SYMPTOMS: Intermittent claudication

RELIEVING FACTORS: None

PAST MEDICAL HISTORY: MI, age 69

FAMILY HISTORY: CVA, father

OBJECTIVE FINDINGS:

VITALS:
LEFT BP: 172/88 mmHg
RESP: 16 cpm
PULSE: 76 bpm
TEMP: 97.9 F

RANGE OF MOTION:
Mildly reduced lumbar ROM.

ORTHOPEDIC TESTING:
Pheasant test produces abdominal pain.

LABORATORY STUDIES:
Elevated LDL-C.

OTHER FINDINGS:
Pedal dusky rubor on dependency.

Part III Handouts v.1.0 Page 36 of 142


Answer the following three (3) questions based on the patient
information provided on the previous page.

30 . Select the three (3) correct diagnoses. (CHOOSE 3)

A. Prostate carcinoma E. Systolic hypertension

B. Abdominal aortic aneurysm F. Spinal stenosis

C. Peripheral artery disease G. Fecal impaction of sigmoid colon

D. Appendicitis H. Cystic kidney disease

31 . Select three (3) risk factors for the patient’s condition shown on CT scan. (CHOOSE 3)

A. Hypertension E. Excessive dietary calcium

B. Hepatic cirrhosis F. Hypercholesterolemia

C. Hirschsprung disease G. Addison disease

D. Cigarette smoking H. Lymphoid hyperplasia

32 . Which of the following is appropriate long term management for this case? (CHOOSE 3)

A. Progressive walking program E. Monitor BP regularly

B. Increase alcohol consumption F. Shoulder range of motion exercises

C. Counsel patient on following DASH diet G. Regular orthopedic testing

D. Regular side posture adjusting H. Encourage limited physical activity

Part III Handouts v.1.0 Page 37 of 142


Answer the following three (3) questions based on the patient
information provided on the previous page.

30 . Select the three (3) correct diagnoses. (CHOOSE 3)

A. Prostate carcinoma E. Systolic hypertension

B. Abdominal aortic aneurysm F. Spinal stenosis

C. Peripheral artery disease G. Fecal impaction of sigmoid colon

D. Appendicitis H. Cystic kidney disease

31 . Select three (3) risk factors for the patient’s condition shown on CT scan. (CHOOSE 3)

A. Hypertension E. Excessive dietary calcium

B. Hepatic cirrhosis F. Hypercholesterolemia

C. Hirschsprung disease G. Addison disease

D. Cigarette smoking H. Lymphoid hyperplasia

32 . Which of the following is appropriate long term management for this case? (CHOOSE 3)

A. Progressive walking program E. Monitor BP regularly

B. Increase alcohol consumption F. Shoulder range of motion exercises

C. Counsel patient on following DASH diet G. Regular orthopedic testing

D. Regular side posture adjusting H. Encourage limited physical activity

Part III Handouts v.1.0 Page 38 of 142


PATIENT INFORMATION

SEX: M AGE: 24

SUBJECTIVE FINDINGS:

CHIEF COMPLAINT: Low back pain

MECHANISM OF INJURY: Unknown; insidious

QUALITY OF PAIN: Achy

SEVERITY OF PAIN: 4/10

AGGRAVATING FACTORS: Rest, lifting

ASSOCIATED SYMPTOMS: Morning stiffness lasting 4 hours

RELIEVING FACTORS: NSAIDs

PAST MEDICAL HISTORY: Unremarkable

FAMILY HISTORY: Unremarkable

OBJECTIVE FINDINGS:

VITALS:
LEFT BP: 112/70 mmHg
RESP: 16 cpm
PULSE: 70 bpm
TEMP: 99.0 F

RANGE OF MOTION:
Lumbar range of motion significantly reduced
in all ranges.

ORTHOPEDIC TESTING:
Positive Yeoman’s test.

LABORATORY STUDIES:
Elevated ESR.

OTHER FINDINGS:
None.

Part III Handouts v.1.0 Page 39 of 142


Answer the following three (3) questions based on the patient
information provided on the previous page.

01 . Select three (3) x-ray findings. (CHOOSE 3)

A. Flowing osteophytes E. Hyperostosis

B. Trolley track sign F. Soft-tissue calcification

C. Porcelain gall bladder G. Loss of disc space

D. Non-marginal syndesmophytes H. Bilateral SI joint ankylosis

02 . Select three (3) additional orthopedic test that would be positive. (CHOOSE 3)

A. Valsalva maneuver E. Shepelmann sign

B. Fajersztajn test F. Adam sign

C. Shober test G. Belt test

D. Sicard test H. Patte test

03 . Select the three (3) most appropriate prognostic outcomes. (CHOOSE 3)

A. Prognosis is poor E. May produce dysphagia

B. Prognosis is good F. Iritis is possible

C. Condition is self-resolving G. Relapsing/remitting course

D. Will progress cephalically H. Poorer prognosis in females

Part III Handouts v.1.0 Page 40 of 142


Answer the following three (3) questions based on the patient
information provided on the previous page.

01 . Select three (3) x-ray findings. (CHOOSE 3)

A. Flowing osteophytes E. Hyperostosis

B. Trolley track sign F. Soft-tissue calcification

C. Porcelain gall bladder G. Loss of disc space

D. Non-marginal syndesmophytes H. Bilateral SI joint ankylosis

02 . Select three (3) additional orthopedic test that would be positive. (CHOOSE 3)

A. Valsalva maneuver E. Shepelmann sign

B. Fajersztajn test F. Adam sign

C. Shober test G. Belt test

D. Sicard test H. Patte test

03 . Select the three (3) most appropriate prognostic outcomes. (CHOOSE 3)

A. Prognosis is poor E. May produce dysphagia

B. Prognosis is good F. Iritis is possible

C. Condition is self-resolving G. Relapsing/remitting course

D. Will progress cephalically H. Poorer prognosis in females

Part III Handouts v.1.0 Page 41 of 142


PATIENT INFORMATION

SEX: M AGE: 24

SUBJECTIVE FINDINGS:

CHIEF COMPLAINT: Left knee pain

MECHANISM OF INJURY: Insidious

QUALITY OF PAIN: Deep, boring

SEVERITY OF PAIN: 7/10

AGGRAVATING FACTORS: Flexion, extension, of knee; walking, climbing stairs

ASSOCIATED SYMPTOMS: Cough

RELIEVING FACTORS: None; worse with rest, at night

PAST MEDICAL HISTORY: Unremarkable

FAMILY HISTORY: Non-contributory

OBJECTIVE FINDINGS:

VITALS:
LEFT BP: 114/76 mmHg
RESP: 20 cpm
PULSE: 88 bpm
TEMP: 99.5 F

RANGE OF MOTION:
Left knee ROM reduced due to pain.

ORTHOPEDIC TESTING:
Ely sign positive on left.

LABORATORY STUDIES:
Elevated alkaline phosphatase.

OTHER FINDINGS:
Tender mass at left distal femur.

Part III Handouts v.1.0 Page 42 of 142


Answer the following three (3) questions based on the patient
information provided on the previous page.

01 . What three (3) additional lab or imaging studies should be performed? (CHOOSE 3)

A. Liver biopsy E. Acid phosphatase

B. HLA-B27 F. Chest x-ray

C. Bone scan G. EMG

D. Right hip x-ray H. Chest CT scan

02 . Choose the three (3) most likely differential diagnoses. (CHOOSE 3)

A. Ewing sarcoma E. Osteosarcoma

B. Reticulum cell sarcoma F. Aneurysmal bone cyst

C. Myositis Ossificans G. Giant cell tumor

D. Fibrosarcoma H. Brodie’s abscess

03 . Choose the three (3) radiological findings seen on the film provided. (CHOOSE 3)

A. Onion skin appearance E. Lobulated appearance

B. Sunburst appearance F. Periosteal reaction

C. Ground glass appearance G. Spiculated appearance

D. Fallen fragment sign H. Shepherd’s crook deformity

Part III Handouts v.1.0 Page 43 of 142


Answer the following three (3) questions based on the patient
information provided on the previous page.

01 . What three (3) additional lab or imaging studies should be performed? (CHOOSE 3)

A. Liver biopsy E. Acid phosphatase

B. HLA-B27 F. Chest x-ray

C. Bone scan G. EMG

D. Right hip x-ray H. Chest CT scan

02 . Choose the three (3) most likely differential diagnoses. (CHOOSE 3)

A. Ewing sarcoma E. Osteosarcoma

B. Reticulum cell sarcoma F. Aneurysmal bone cyst

C. Myositis Ossificans G. Giant cell tumor

D. Fibrosarcoma H. Brodie’s abscess

03 . Choose the three (3) radiological findings seen on the film provided. (CHOOSE 3)

A. Onion skin appearance E. Lobulated appearance

B. Sunburst appearance F. Periosteal reaction

C. Ground glass appearance G. Spiculated appearance

D. Fallen fragment sign H. Shepherd’s crook deformity

Part III Handouts v.1.0 Page 44 of 142


PATIENT INFORMATION

SEX: F AGE: 38

SUBJECTIVE FINDINGS:

CHIEF COMPLAINT: Ptosis bilaterally.

MECHANISM OF INJURY: Developed after Salmonella enteritis.

QUALITY OF PAIN: None.

SEVERITY OF PAIN: N/A

AGGRAVATING FACTORS: Ptosis worsens as day progresses; worst in evening.

ASSOCIATED SYMPTOMS: Muscle weakness after activity; improves with rest.

RELIEVING FACTORS: Rest, sleep.

PAST MEDICAL HISTORY: Thyrotoxicosis; resolved with treatment.

FAMILY HISTORY: Mother, 68, has hypothyroidism.

OBJECTIVE FINDINGS:

VITALS:
LEFT BP: 110/66 mmHg
RESP: 14 cpm
PULSE: 74 bpm
TEMP: 98.0 ºF

RANGE OF MOTION:
Unremarkable.

ORTHOPEDIC TESTING:
Repetitive squats produces +3 hip girdle weakness.

LABORATORY STUDIES:
Serum vitamin B12 normal.

OTHER FINDINGS:
Slurred speech with prolonged speaking.

Part III Handouts v.1.0 Page 45 of 142


Answer the following three (3) questions based on the patient
information provided on the previous page.

01 . Which of the following additional lab tests or imaging should be ordered? (CHOOSE 3)

A. ANA E. Cervical spine x-rays

B. CBC F. Chest CT

C. KUB G. ECG

D. HLA-B27 H. Serum electrolytes

02 . Select three correct precipitating factors for this patient’s condition. (CHOOSE 3)

A. Smoking E. TB

B. Hyperthyroidism F. Meningioma

C. Asbestos exposure G. Infection

D. Surgery H. Alcoholism

03 . What additional clinical findings can be expected in this condition? (CHOOSE 3)

A. Dysphagia/choking E. Malar rash

B. Dullness on lung percussion F. Normal DTRs

C. Respiratory failure G. Arm pain

D. Hearing deficit H. Dermatomal numbness

Part III Handouts v.1.0 Page 46 of 142


Answer the following three (3) questions based on the patient
information provided on the previous page.

01 . Which of the following additional lab tests or imaging should be ordered? (CHOOSE 3)

A. ANA E. Cervical spine x-rays

B. CBC F. Chest CT

C. KUB G. ECG

D. HLA-B27 H. Serum electrolytes

02 . Select three correct precipitating factors for this patient’s condition. (CHOOSE 3)

A. Smoking E. TB

B. Hyperthyroidism F. Meningioma

C. Asbestos exposure G. Infection

D. Surgery H. Alcoholism

03 . What additional clinical findings can be expected in this condition? (CHOOSE 3)

A. Dysphagia/choking E. Malar rash

B. Dullness on lung percussion F. Normal DTRs

C. Respiratory failure G. Arm pain

D. Hearing deficit H. Dermatomal numbness

Part III Handouts v.1.0 Page 47 of 142


PATIENT INFORMATION

SEX: M AGE: 27

SUBJECTIVE FINDINGS:

CHIEF COMPLAINT: Left upper quadrant abdominal discomfort

MECHANISM OF INJURY: None.

QUALITY OF PAIN: Pressure, fullness, mild ache

SEVERITY OF PAIN: 1-2/10

AGGRAVATING FACTORS: Palpation in LUQ

ASSOCIATED SYMPTOMS: Can’t take deep breaths; dysphagia with reflux

RELIEVING FACTORS: None

PAST MEDICAL HISTORY: Pharyngitis 3 months ago

FAMILY HISTORY: Non-contributory

OBJECTIVE FINDINGS:

VITALS:
LEFT BP: 110/64 mmHg
RESP: 20 cpm
PULSE: 82 bpm
TEMP: 99.6 ºF

RANGE OF MOTION:
Thoracic range of motion normal.

ORTHOPEDIC TESTING:
N/A

LABORATORY STUDIES:
Total cholesterol: 267 mg/dL.

OTHER FINDINGS:
Cervical lymphadenopathy.

Part III Handouts v.1.0 Page 48 of 142


Answer the following three (3) questions based on the patient
information provided on the previous page.

01 . Select the appropriate additional lab tests needed in this case. (CHOOSE 3)

A. Serum fasting glucose E. Epstein-Barr virus antibody

B. WBC count with differential F. Mono-spot test

C. Bone marrow biopsy G. Throat culture

D. Lymph node biopsy H. Urinalysis

02 . What additional clinical findings are expected in this case? (CHOOSE 3)

A. Pericardial friction rub E. Urinary tract infection

B. Fatigue F. Murphy’s punch positive

C. Hepatomegaly G. Productive cough

D. Peripheral edema H. Splenomegaly

03 . Which of the following clinical impressions/diagnoses apply? (CHOOSE 3)

A. Rheumatic fever E. Sickle-cell disease

B. Chronic fatigue syndrome F. Gastric ulcer

C. Infectious mononucleosis G. Hypercholesterolemia

D. Plasmacytoid lymphocyte H. Non-Hodgkins lymphoma

Part III Handouts v.1.0 Page 49 of 142


Answer the following three (3) questions based on the patient
information provided on the previous page.

01 . Select the appropriate additional lab tests needed in this case. (CHOOSE 3)

A. Serum fasting glucose E. Epstein-Barr virus antibody

B. WBC count with differential F. Mono-spot test

C. Bone marrow biopsy G. Throat culture

D. Lymph node biopsy H. Urinalysis

02 . What additional clinical findings are expected in this case? (CHOOSE 3)

A. Pericardial friction rub E. Urinary tract infection

B. Fatigue F. Murphy’s punch positive

C. Hepatomegaly G. Productive cough

D. Peripheral edema H. Splenomegaly

03 . Which of the following clinical impressions/diagnoses apply? (CHOOSE 3)

A. Rheumatic fever E. Sickle-cell disease

B. Chronic fatigue syndrome F. Gastric ulcer

C. Infectious mononucleosis G. Hypercholesterolemia

D. Plasmacytoid lymphocyte H. Non-Hodgkins lymphoma

Part III Handouts v.1.0 Page 50 of 142


PATIENT INFORMATION

SEX: F AGE: 54

SUBJECTIVE FINDINGS:

CHIEF COMPLAINT: Pain in right lateral forefoot.

MECHANISM OF INJURY: Sudden, while walking 5 miles to work.

QUALITY OF PAIN: Cramp-like.

SEVERITY OF PAIN: 5/10.

AGGRAVATING FACTORS: Standing, walking, palpation.

ASSOCIATED SYMPTOMS: Edema of foot and ankle.

RELIEVING FACTORS: Rest, elevation of foot.

PAST MEDICAL HISTORY: Premature menopause (age 39).

FAMILY HISTORY: Non-contributory.

OBJECTIVE FINDINGS:

VITALS:
LEFT BP: 135/88 mmHg
RESP: 16 cpm
PULSE: 77 bpm
TEMP: 97.6 ºF

RANGE OF MOTION:
ROM of toes and ankle WNL.

ORTHOPEDIC TESTING:
Metatarsal squeeze produces pain in
right lateral forefooot.

LABORATORY STUDIES:
None.

OTHER FINDINGS:
Superficial lower leg varicosities bilaterally.

Part III Handouts v.1.0 Page 51 of 142


Answer the following three (3) questions based on the patient
information provided on the previous page.

01 . What additional lab tests or imaging is needed? (CHOOSE 3)

A. CBC E. Pituitary function tests

B. Doppler ultrasound of lower leg veins F. Chest x-ray

C. DEXA G. Serum fasting glucose

D. Serum electrolytes H. Foot radiographs

02 . Select the 3 correct clinical impressions for this case. (CHOOSE 3)

A. Suspected hypopituitarism E. Suspected March fracture

B. Atherosclerotic vascular disease F. Plantar fasciitis

C. Chronic venous insufficiency G. Morton’s neuroma

D. Suspected osteoporosis H. Suspected diabetes mellitus, type 2

03 . What is appropriate initial case management for this patient? (CHOOSE 3)

A. Eliminate weight bearing on affected foot E. Referral to neurosurgeon

B. Ultrasound underwater to foot F. Referral to orthopedic surgeon

C. Manipulate metatarsals G. Supplementation with calcium & vitamin


D

D. Plantar fascia stripping H. Low glycemic diet

Part III Handouts v.1.0 Page 52 of 142


Answer the following three (3) questions based on the patient
information provided on the previous page.

01 . What additional lab tests or imaging is needed? (CHOOSE 3)

A. CBC E. Pituitary function tests

B. Doppler ultrasound of lower leg veins F. Chest x-ray

C. DEXA G. Serum fasting glucose

D. Serum electrolytes H. Foot radiographs

02 . Select the 3 correct clinical impressions for this case. (CHOOSE 3)

A. Suspected hypopituitarism E. Suspected March fracture

B. Atherosclerotic vascular disease F. Plantar fasciitis

C. Chronic venous insufficiency G. Morton’s neuroma

D. Suspected osteoporosis H. Suspected diabetes mellitus, type 2

03 . What is appropriate initial case management for this patient? (CHOOSE 3)

A. Eliminate weight bearing on affected foot E. Referral to neurosurgeon

B. Ultrasound underwater to foot F. Referral to orthopedic surgeon

C. Manipulate metatarsals G. Supplementation with calcium & vitamin D

D. Plantar fascia stripping H. Low glycemic diet

Part III Handouts v.1.0 Page 53 of 142


PATIENT INFORMATION

SEX: M AGE: 43

SUBJECTIVE FINDINGS:

CHIEF COMPLAINT: Recurrent headaches over right eye

MECHANISM OF INJURY: None

QUALITY OF PAIN: Sharp ache, stabbing

SEVERITY OF PAIN: severe

AGGRAVATING FACTORS: alcohol

ASSOCIATED SYMPTOMS: Ipsilateral ptosis and forehead sweating

RELIEVING FACTORS: self-limiting in about 1 hour

PAST MEDICAL HISTORY: Tonsillectomy, age 6

FAMILY HISTORY: patient is adopted

OBJECTIVE FINDINGS:

VITALS:
LEFT BP: 138/98 mmHg
RESP: 20 cpm
PULSE: 86 bpm
TEMP: 97.8ºF

RANGE OF MOTION:
Normal cervical ROM

ORTHOPEDIC TESTING:
Brudzinski negative

LABORATORY STUDIES:
RBC count elevated

OTHER FINDINGS:
Cough due to smoking

Part III Handouts v.1.0 Page 54 of 142


Answer the following three (3) questions based on the patient
information provided on the previous page.

01 . Which of the following includes appropriate long term management? (CHOOSE 3)

A. Avoid alcohol E. Stop smoking

B. Anti-histamines F. Meditation

C. Avoid allergens G. Melatonin

D. Short-wave diathermy over sinuses H. Aerobic exercise

02 . Select the appropriate diagnoses for this case. (CHOOSE 3)

A. Leukemia E. Sinus headache

B. Secondary polycythemia F. Hypertension

C. Migraine headache G. Pre-hypertension

D. Cluster headache H. Iritis

03 . Select appropriate risk factors for this headache. (CHOOSE 3)

A. Male E. Female

B. Family history F. Meningitis

C. High salt intake G. Diabetes mellitus

D. High sugar intake H. Smoking

Part III Handouts v.1.0 Page 55 of 142


Answer the following three (3) questions based on the patient
information provided on the previous page.

01 . Which of the following includes appropriate long term management? (CHOOSE 3)

A. Avoid alcohol E. Stop smoking

B. Anti-histamines F. Meditation

C. Avoid allergens G. Melatonin

D. Short-wave diathermy over sinuses H. Aerobic exercise

02 . Select the appropriate diagnoses for this case. (CHOOSE 3)

A. Leukemia E. Sinus headache

B. Secondary polycythemia F. Hypertension

C. Migraine headache G. Pre-hypertension

D. Cluster headache H. Iritis

03 . Select appropriate risk factors for this headache. (CHOOSE 3)

A. Male E. Female

B. Family history F. Meningitis

C. High salt intake G. Diabetes mellitus

D. High sugar intake H. Smoking

Part III Handouts v.1.0 Page 56 of 142


PATIENT INFORMATION

SEX: M AGE: 64

SUBJECTIVE FINDINGS:

CHIEF COMPLAINT: Bilateral leg pain

MECHANISM OF INJURY: None

QUALITY OF PAIN: Heaviness, aching, tingling, numbness

SEVERITY OF PAIN: Mild-to-moderate

AGGRAVATING FACTORS: Walking, standing, swimming

ASSOCIATED SYMPTOMS: Urinary hesitancy, urgency, frequency, dribbling

RELIEVING FACTORS: Sitting down to rest

PAST MEDICAL HISTORY: Multiple disc herniations

FAMILY HISTORY: Father: MI and CVA

OBJECTIVE FINDINGS:

VITALS:
LEFT BP: 130/86 mmHg
RESP: 18 cpm
PULSE: 78 bpm
TEMP: 98.5ºF

RANGE OF MOTION:
Lumbar ROM restricted globally

ORTHOPEDIC TESTING:
Stoop test positive

LABORATORY STUDIES:
Urinalysis and routine chemistry panel WNL

OTHER FINDINGS:
Impaired vibratory sense bilaterally in feet

Part III Handouts v.1.0 Page 57 of 142


Answer the following three (3) questions based on the patient
information provided on the previous page.

01 . Select the most appropriate diagnoses for this patient. (CHOOSE 3)

A. Diabetes mellitus E. Acute disc herniation

B. UTI F. Prostatic hypertrophy

C. Vascular claudication G. Neurogenic claudication

D. Central canal stenosis H. Deep vein thrombosis

02 . Which of the additional clinical findings can be expected? (CHOOSE 3)

A. Bruit E. Hyporeflexia in the legs

B. Diminished peripheral pulses F. Babinski sign

C. Elevated plasma glucose G. Diminished pinprick sensation in feet

D. Elevated PSA H. Positive Homan sign

03 . Which of the following are appropriate for long term management? (CHOOSE 3)

A. Flexion-Distraction technique E. McKenzie extension exercises

B. Niacin F. Weekly ultrasound to low back

C. Williams exercises G. Abdominal strengthening

D. Low glycemic diet H. Bed rest

Part III Handouts v.1.0 Page 58 of 142


Answer the following three (3) questions based on the patient
information provided on the previous page.

01 . Select the most appropriate diagnoses for this patient. (CHOOSE 3)

A. Diabetes mellitus E. Acute disc herniation

B. UTI F. Prostatic hypertrophy

C. Vascular claudication G. Neurogenic claudication

D. Central canal stenosis H. Deep vein thrombosis

02 . Which of the additional clinical findings can be expected? (CHOOSE 3)

A. Bruit E. Hyporeflexia in the legs

B. Diminished peripheral pulses F. Babinski sign

C. Elevated plasma glucose G. Diminished pinprick sensation in feet

D. Elevated PSA H. Positive Homan sign

03 . Which of the following are appropriate for long term management? (CHOOSE 3)

A. Flexion-Distraction technique E. McKenzie extension exercises

B. Niacin F. Weekly ultrasound to low back

C. Williams exercises G. Abdominal strengthening

D. Low glycemic diet H. Bed rest

Part III Handouts v.1.0 Page 59 of 142


PATIENT INFORMATION

SEX: F AGE: 69

SUBJECTIVE FINDINGS:

CHIEF COMPLAINT: Acute low back pain.

MECHANISM OF INJURY: Bending to tie shoes.

QUALITY OF PAIN: Ache, sharp at times.

SEVERITY OF PAIN: Severe.

AGGRAVATING FACTORS: Upright position, flexion.

ASSOCIATED SYMPTOMS: Constipation.

RELIEVING FACTORS: Recumbency.

PAST MEDICAL HISTORY: Exertional dyspnea for last 12 years from heavy smoking.

FAMILY HISTORY: Mother, stroke.

OBJECTIVE FINDINGS:

VITALS:
LEFT BP: 140/95 mmHg
RESP: 20 cpm
PULSE: 88 bpm
TEMP: 98.4 ºF

RANGE OF MOTION:
Lumbar range of motion severely restricted.

ORTHOPEDIC TESTING:
Unable to perform due to pain.

LABORATORY STUDIES:
Normal serum calcium.

OTHER FINDINGS:
Tympanic abdomen with < 5 bowel sounds/min.
Increased AP chest diameter.

Part III Handouts v.1.0 Page 60 of 142


Answer the following three (3) questions based on the patient
information provided on the previous page.

01 . Select the radiologic findings that can be seen on the x-ray. (CHOOSE 3)

A. Traumatic compression fracture E. Post-menopausal osteoporotic


compression fracture

B. Pathologic compression fracture F. Decreased bone density

C. Retrolisthesis of L4 G. Anterolisthesis of L5

D. Benign tumor H. Hyperlordosis

02 . Select the three other diagnosis that this patient has. (CHOOSE 3)

A. Adynamic ileus E. Blastic metastasis

B. Metastatic breast cancer F. Multiple myeloma

C. Hyperparathyroidism G. Platyspondyly

D. Hypertension H. COPD

03 . Which of the following is appropriate initial case management? (CHOOSE 3)

A. Lumbar brace E. Cox flexion-distraction

B. Spinal manipulation F. Ultrasound

C. Refer to oncologist G. Referral for pain control

D. Vitamin D and calcium H. Extension exercises

Part III Handouts v.1.0 Page 61 of 142


Answer the following three (3) questions based on the patient
information provided on the previous page.

01 . Select the radiologic findings that can be seen on the x-ray. (CHOOSE 3)

A. Traumatic compression fracture E. Post-menopausal osteoporotic compression


fracture

B. Pathologic compression fracture F. Decreased bone density

C. Retrolisthesis of L4 G. Anterolisthesis of L5

D. Benign tumor H. Hyperlordosis

02 . Select the three other diagnosis that this patient has. (CHOOSE 3)

A. Adynamic ileus E. Blastic metastasis

B. Metastatic breast cancer F. Multiple myeloma

C. Hyperparathyroidism G. Platyspondyly

D. Hypertension H. COPD

03 . Which of the following is appropriate initial case management? (CHOOSE 3)

A. Lumbar brace E. Cox flexion-distraction

B. Spinal manipulation F. Ultrasound

C. Refer to oncologist G. Referral for pain control

D. Vitamin D and calcium H. Extension exercises

Part III Handouts v.1.0 Page 62 of 142


PATIENT INFORMATION

SEX: F AGE: 35

SUBJECTIVE FINDINGS:

CHIEF COMPLAINT: Urinary urgency, frequency.

MECHANISM OF INJURY: None.

QUALITY OF PAIN: None.

SEVERITY OF PAIN: N/A.

AGGRAVATING FACTORS: Drinking a lot of coffee.

ASSOCIATED SYMPTOMS: Constipation.

RELIEVING FACTORS: None.

PAST MEDICAL HISTORY: Diagnosed with SLE at age 26.

FAMILY HISTORY: Mother, 74, has diabetes mellitus.

OBJECTIVE FINDINGS:

VITALS:
LEFT BP: 144/94 mmHg
RESP: 18 cpm
PULSE: 86 bpm
TEMP: 98.8 ºF

RANGE OF MOTION:
N/A.

ORTHOPEDIC TESTING:
N/A.

LABORATORY STUDIES:
BUN, creatinine, ESR WNL.

OTHER FINDINGS:
Murphy’s punch negative bilaterally.

Part III Handouts v.1.0 Page 63 of 142


Answer the following three (3) questions based on the patient
information provided on the previous page.

01 . What additional lab tests or imaging is needed in this case? (CHOOSE 3)

A. Lumbar x-rays E. Pelvic ultrasound

B. HbA1c F. Colonoscopy

C. PSA G. Serum electrolytes

D. UA H. ANA

02 . Select the most appropriate differential diagnoses for this patient. (CHOOSE 3)

A. Interstitial cystitis E. Pyelonephritis

B. Lupus nephropathy F. Hypercalcemia

C. Diabetes mellitus G. Proctitis

D. Chronic renal failure H. Endometriosis

03 . What is the long-term management for the most likely diagnosis? (CHOOSE 3)

A. Referral to rheumatologist E. Referral for antibiotics

B. Weight loss F. Low glycemic diet

C. Referral to urologist G. Adjust atlas

D. Referral to endocrinologist H. Vegetarian diet

Part III Handouts v.1.0 Page 64 of 142


Answer the following three (3) questions based on the patient
information provided on the previous page.

01 . What additional lab tests or imaging is needed in this case? (CHOOSE 3)

A. Lumbar x-rays E. Pelvic ultrasound

B. HbA1c F. Colonoscopy

C. PSA G. Serum electrolytes

D. UA H. ANA

02 . Select the most appropriate differential diagnoses for this patient. (CHOOSE 3)

A. Interstitial cystitis E. Pyelonephritis

B. Lupus nephropathy F. Hypercalcemia

C. Diabetes mellitus G. Proctitis

D. Chronic renal failure H. Endometriosis

03 . What is the long-term management for the most likely diagnosis? (CHOOSE 3)

A. Referral to rheumatologist E. Referral for antibiotics

B. Weight loss F. Low glycemic diet

C. Referral to urologist G. Adjust atlas

D. Referral to endocrinologist H. Vegetarian diet

Part III Handouts v.1.0 Page 65 of 142


PATIENT INFORMATION

SEX: M AGE: 77

SUBJECTIVE FINDINGS:

CHIEF COMPLAINT: Cough and chest pain on the right.

MECHANISM OF INJURY: None.

QUALITY OF PAIN: Sharp, stabbing.

SEVERITY OF PAIN: Moderately severe.

AGGRAVATING FACTORS: Deep breathing, moving trunk.

ASSOCIATED SYMPTOMS: Purulent, blood-streaked sputum production.

RELIEVING FACTORS: Shallow breathing, rest.

PAST MEDICAL HISTORY: Post-traumatic splenectomy, 1999.

FAMILY HISTORY: Non-contributory.

OBJECTIVE FINDINGS:

VITALS:
LEFT BP: 155/90 mmHg
RESP: 26 cpm
PULSE: 120 bpm
TEMP: 102.4 ºF

RANGE OF MOTION:
Restricted and painful thoracic
flexion and extension.

ORTHOPEDIC TESTING:
Schepelmann’s sign: pain on the
right when bending to the left.

LABORATORY STUDIES:
WBC count: 13500
Neutrophils: 77%
Lymphocytes: 12%

OTHER FINDINGS:
Muscle splinting in right upper-to-mid
rib cage.

Part III Handouts v.1.0 Page 66 of 142


Answer the following three (3) questions based on the patient
information provided on the previous page.
KEY

04 . What additional clinical findings are expected in this case? (CHOOSE 3)

A. Decreased tactile fremitus E. Positive Soto-Hall

B. Dullness on percussion F. Pericardial friction rub

C. Hyperresonance on percussion G. Crackles

D. Egophony H. Flow murmur

05 . Which radiology findings/impressions are present in the film provided? (CHOOSE 3)

A. Costophrenic angle blunting E. Lung cancer

B. Cardiomegaly F. Granuloma

C. Lobar consolidation G. Normal left lung

D. Hiatal hernia H. Pneumothorax

06 . Which of the following diagnoses apply to this case? (CHOOSE 3)

A. Congestive heart failure E. Pleural effusion

B. Pneumococcal pneumonia F. Tuberculosis

C. Coxackievirus pneumonia G. Hypertension

D. Myoplasma pneumonia H. Acute pericarditis

Part III Handouts v.1.0 Page 67 of 142


Answer the following three (3) questions based on the patient
information provided on the previous page.

04 . What additional clinical findings are expected in this case? (CHOOSE 3)

A. Decreased tactile fremitus E. Positive Soto-Hall

B. Dullness on percussion F. Pericardial friction rub

C. Hyperresonance on percussion G. Crackles

D. Egophony H. Flow murmur

05 . Which radiology findings/impressions are present in the film provided? (CHOOSE 3)

A. Costophrenic angle blunting E. Lung cancer

B. Cardiomegaly F. Granuloma

C. Lobar consolidation G. Normal left lung

D. Hiatal hernia H. Pneumothorax

06 . Which of the following diagnoses apply to this case? (CHOOSE 3)

A. Congestive heart failure E. Pleural effusion

B. Pneumococcal pneumonia F. Tuberculosis

C. Coxackievirus pneumonia G. Hypertension

D. Myoplasma pneumonia H. Acute pericarditis

Part III Handouts v.1.0 Page 68 of 142


PATIENT INFORMATION

SEX: F AGE: 30

SUBJECTIVE FINDINGS:

CHIEF COMPLAINT: Headache

MECHANISM OF INJURY: Insidious

QUALITY OF PAIN: Throbbing

SEVERITY OF PAIN: 5/10

AGGRAVATING FACTORS: Exertion, straining, recumbent position

ASSOCIATED SYMPTOMS: Weight gain, polyuria

RELIEVING FACTORS: Rest in sitting position

PAST MEDICAL HISTORY: Unremarkable

FAMILY HISTORY: Duodenal ulcer, father

OBJECTIVE FINDINGS:

VITALS:
LEFT BP: 190/114 mmHg
RESP: 18 cpm
PULSE: 84 bpm
TEMP: 97.5 F

RANGE OF MOTION:
Unremarkable.

ORTHOPEDIC TESTING:
Valsalva aggravates headache.

LABORATORY STUDIES:
Elevated serum sodium,
Decreased serum potassium.

OTHER FINDINGS:
Abdominal straie

Part III Handouts v.1.0 Page 69 of 142


Answer the following three (3) questions based on the patient
information provided on the previous page.

01 . Choose three (3) other lab or imaging studies required in this case. (CHOOSE 3)

A. Fasting blood glucose E. Thyroid panel

B. Abdominal MRI F. Urinary 5-HIAA

C. 24 hour urinary cortisol G. Bone scan

D. Upper GI study H. Liver function tests

02 . Select three (3) complications that can be expected with this condition. (CHOOSE 3)

A. Rheumatoid arthritis E. Osteosarcoma

B. Hypothyroidism F. Osteoporosis

C. Diabetes mellitus G. Deafness

D. Malabsorption syndrome H. Renal calculi

03 . Select the three (3) possible etiologic organs for this condition. (CHOOSE 3)

A. Kidney E. Heart

B. Anterior pituitary F. Liver

C. Posterior pituitary G. Lung

D. Adrenal gland H. Brain

Part III Handouts v.1.0 Page 70 of 142


Answer the following three (3) questions based on the patient
information provided on the previous page.

01 . Choose three (3) other lab or imaging studies required in this case. (CHOOSE 3)

A. Fasting blood glucose E. Thyroid panel

B. Abdominal MRI F. Urinary 5-HIAA

C. 24 hour urinary cortisol G. Bone scan

D. Upper GI study H. Liver function tests

02 . Select three (3) complications that can be expected with this condition. (CHOOSE 3)

A. Rheumatoid arthritis E. Osteosarcoma

B. Hypothyroidism F. Osteoporosis

C. Diabetes mellitus G. Deafness

D. Malabsorption syndrome H. Renal calculi

03 . Select the three (3) possible etiologic organs for this condition. (CHOOSE 3)

A. Kidney E. Heart

B. Anterior pituitary F. Liver

C. Posterior pituitary G. Lung

D. Adrenal gland H. Brain

Part III Handouts v.1.0 Page 71 of 142


PATIENT INFORMATION

SEX: F AGE: 31

SUBJECTIVE FINDINGS:

CHIEF COMPLAINT: Suprapubic pain when urinating

MECHANISM OF INJURY: Insidious 2 months ago

QUALITY OF PAIN: Burning

SEVERITY OF PAIN: Moderate

AGGRAVATING FACTORS: Urination, palpation in suprapubic area

ASSOCIATED SYMPTOMS: Dysmenorrhea, dyspareunia

RELIEVING FACTORS: None

PAST MEDICAL HISTORY: First onset of dysmenorrhea at age 27

FAMILY HISTORY: Non-contributory

OBJECTIVE FINDINGS:

VITALS:
LEFT BP: 114/68 mmHg
RESP: 16 cpm
PULSE: 76 bpm
TEMP: 97.9 F

RANGE OF MOTION:
Lumbar ROM within normal limits.

ORTHOPEDIC TESTING:
Patrick FABERE negative bilaterally.

LABORATORY STUDIES:
RBC count low, MCV low, hemoglobin low;
Ferritin low.
UA: negative for nitrites, leukocyte esterase
and blood.

OTHER FINDINGS:
Suprapubic tenderness.

Part III Handouts v.1.0 Page 72 of 142


Answer the following three (3) questions based on the patient
information provided on the previous page.

01 . Which of the following need to be included in the diagnosis? (CHOOSE 3)

A. Iron deficiency E. Microcytic anemia

B. Urinary tract infection F. Macrocytic anemia

C. Urolithiasis G. Pelvic inflammatory disease

D. Endometriosis H. Proctitis

02 . What additional studies should be performed on this patient? (CHOOSE 3)

A. Gynecological exam E. Serum iron

B. Pelvic MRI F. Serum folate

C. O & P G. Aortic angiography

D. Laparoscopic biopsy H. Inguinal hernia exam

03 . What additional signs or symptoms may be seen with this condition? (CHOOSE 3)

A. Colonic polyps E. Lymphadenopathy

B. Palpable pelvic mass F. Uterine fibroids

C. Abdominal bloating G. Infertility

D. Rheumatoid arthritis H. Bladder carcinoma

Part III Handouts v.1.0 Page 73 of 142


Answer the following three (3) questions based on the patient
information provided on the previous page.

01 . Which of the following need to be included in the diagnosis? (CHOOSE 3)

A. Iron deficiency E. Microcytic anemia

B. Urinary tract infection F. Macrocytic anemia

C. Urolithiasis G. Pelvic inflammatory disease

D. Endometriosis H. Proctitis

02 . What additional studies should be performed on this patient? (CHOOSE 3)

A. Gynecological exam E. Serum iron

B. Pelvic MRI F. Serum folate

C. O & P G. Aortic angiography

D. Laparoscopic biopsy H. Inguinal hernia exam

03 . What additional signs or symptoms may be seen with this condition? (CHOOSE 3)

A. Colonic polyps E. Lymphadenopathy

B. Palpable pelvic mass F. Uterine fibroids

C. Abdominal bloating G. Infertility

D. Rheumatoid arthritis H. Bladder carcinoma

Part III Handouts v.1.0 Page 74 of 142


PATIENT INFORMATION

SEX: M AGE: 67

SUBJECTIVE FINDINGS:

CHIEF COMPLAINT: Midback pain

MECHANISM OF INJURY: Insidious

QUALITY OF PAIN: Stiff, achy

SEVERITY OF PAIN: Mild

AGGRAVATING FACTORS: Lying spine

ASSOCIATED SYMPTOMS: Exertional dyspnea, morning cough

RELIEVING FACTORS: Resting in lounge chair

PAST MEDICAL HISTORY: Lumbar disc herniation, age 33

FAMILY HISTORY: Father, emphysema

OBJECTIVE FINDINGS:

VITALS:
LEFT BP: 158/94
RESP: 20 cpm
PULSE: 90 bpm, irregular
TEMP: 98.4ºF

RANGE OF MOTION:
Decreased cervical flexion

ORTHOPEDIC TESTING:
None performed

LABORATORY STUDIES:
None performed

OTHER FINDINGS:
Upper thoracic extension absent

Part III Handouts v.1.0 Page 75 of 142


Answer the following three (3) questions based on the patient
information provided on the previous page.

01 . Select the 3 other expected findings in this case. (CHOOSE 3)

A. Prolonged expiration E. Scoliosis

B. Thoracic intersegmental fixations F. Murmur

C. Hyperresonant percussion G. Epigastric tenderness

D. Anemia H. Pleural friction rub

02 . What additional lab or imaging is needed in this case? (CHOOSE 3)

A. Hgb A1c E. Electrocardiogram

B. Sputum culture F. Cervical spine radiographs

C. Blood culture G. Bone densitometry

D. Pulmonary function tests H. Serum electrolytes

03 . Select the appropriate radiology findings. (CHOOSE 3)

A. Aortic aneurysm E. Enlarged pulmonary arteries

B. Increased AP chest diameter F. Flattening of the diaphragm

C. Gynecomastia G. Sternal fracture

D. Blunted costophrenic sulci H. Mitral annular calcification

Part III Handouts v.1.0 Page 76 of 142


Answer the following three (3) questions based on the patient
information provided on the previous page.

01 . Select the 3 other expected findings in this case. (CHOOSE 3)

A. Prolonged expiration E. Scoliosis

B. Thoracic intersegmental fixations F. Murmur

C. Hyperresonant percussion G. Epigastric tenderness

D. Anemia H. Pleural friction rub

02 . What additional lab or imaging is needed in this case? (CHOOSE 3)

A. Hgb A1c E. Electrocardiogram

B. Sputum culture F. Cervical spine radiographs

C. Blood culture G. Bone densitometry

D. Pulmonary function tests H. Serum electrolytes

03 . Select the appropriate radiology findings. (CHOOSE 3)

A. Aortic aneurysm E. Enlarged pulmonary arteries

B. Increased AP chest diameter F. Flattening of the diaphragm

C. Gynecomastia G. Sternal fracture

D. Blunted costophrenic sulci H. Mitral annular calcification

Part III Handouts v.1.0 Page 77 of 142


PATIENT INFORMATION

SEX: M AGE: 29

SUBJECTIVE FINDINGS:

CHIEF COMPLAINT: Elbow, wrist, knee and ankle pain, and chills.

MECHANISM OF INJURY: None; insidious.

QUALITY OF PAIN: Achy.

SEVERITY OF PAIN: Mild.

AGGRAVATING FACTORS: None.

ASSOCIATED SYMPTOMS: Fatigue, weight loss, left flank pain.

RELIEVING FACTORS: Aleve.

PAST MEDICAL HISTORY: Mitral valve prolapse, chlamydial urethritis.

FAMILY HISTORY: Non-contributory.

OBJECTIVE FINDINGS:

VITALS:
LEFT BP: 120/80 mmHg
RESP: 20 cpm
PULSE: 92 bpm
TEMP: 100.8 ºF

RANGE OF MOTION:
Affected joints have painful ROM but no
restrictions.

ORTHOPEDIC TESTING:
None.

LABORATORY STUDIES:
WBC count: 13400.

OTHER FINDINGS:
Costovertebral angle tenderness on the left.
Mitral holosystolic murmur.

Part III Handouts v.1.0 Page 78 of 142


Answer the following three (3) questions based on the patient
information provided on the previous page.

01 . What additional clinical findings can expected in this case? (CHOOSE 3)

A. Splinter hemorrahges E. Erythema marginatum

B. Subcutaneous nodules F. Night sweats

C. Tophi G. Melena

D. Splenomegaly H. Positive Lachman test

02 . What additional lab tests or imaging is required in this case? (CHOOSE 3)

A. CBC E. Serum electrolytes

B. Blood cultures F. Bone scan

C. UA G. Echocardiogram

D. Liver function tests H. MRI of chest

03 . What complications or sequelae can be expected in this case? (CHOOSE 3)

A. Rheumatoid arthritis E. Renal infarction

B. Nephrolithiasis F. Congestive heart failure

C. Fatal GI bleeding G. Hepatoma

D. Stroke H. Glaucoma

Part III Handouts v.1.0 Page 79 of 142


Answer the following three (3) questions based on the patient
information provided on the previous page.

01 . What additional clinical findings can expected in this case? (CHOOSE 3)

A. Splinter hemorrahges E. Erythema marginatum

B. Subcutaneous nodules F. Night sweats

C. Tophi G. Melena

D. Splenomegaly H. Positive Lachman test

02 . What additional lab tests or imaging is required in this case? (CHOOSE 3)

A. CBC E. Serum electrolytes

B. Blood cultures F. Bone scan

C. UA G. Echocardiogram

D. Liver function tests H. MRI of chest

03 . What complications or sequelae can be expected in this case? (CHOOSE 3)

A. Rheumatoid arthritis E. Renal infarction

B. Nephrolithiasis F. Congestive heart failure

C. Fatal GI bleeding G. Hepatoma

D. Stroke H. Glaucoma

Part III Handouts v.1.0 Page 80 of 142


PATIENT INFORMATION

SEX: M AGE: 44

SUBJECTIVE FINDINGS:

CHIEF COMPLAINT: Weight loss.

MECHANISM OF INJURY: None.

QUALITY OF PAIN: None.

SEVERITY OF PAIN: Not applicable.

AGGRAVATING FACTORS: Reduced food intake.

ASSOCIATED SYMPTOMS: Productive cough.

RELIEVING FACTORS: None.

PAST MEDICAL HISTORY: Excessive high-calorie food intake

FAMILY HISTORY: Asthma, brother.

OBJECTIVE FINDINGS:

VITALS:
LEFT BP: 115/70 mmHg
RESP: 22 cpm
PULSE: 90 bpm
TEMP: 99.9 F

RANGE OF MOTION:
Not relevant.

ORTHOPEDIC TESTING:
Adam’s test negative.

LABORATORY STUDIES:
Elevated WBC count, 14,000
Elevated PMN and monocyte count
PPD positive, 22 mm

OTHER FINDINGS:
Crackles in right upper lobe.

Part III Handouts v.1.0 Page 81 of 142


Answer the following three (3) questions based on the patient
information provided on the previous page.

01 . Select the three (3) most common extra-pulmonary sites of involvement. (CHOOSE 3)

A. Kidney E. Penis

B. Vertebrae F. skin

C. Lymph nodes G. Small intestine

D. Eye H. Brain

02 . Which of the following are risk factors for the condition presented. (CHOOSE 3)

A. Bronchial asthma E. HIV infection

B. Alcoholism F. Hyperthyroidism

C. Rib fractures G. Diabetes mellitus

D. Vegetarianism H. Congestive heart failure

03 . What additional clinical findings can be expected to appear? (CHOOSE 3)

A. Rhinitis E. Diarrhea

B. Constipation F. Anorexia

C. Hearing loss G. Pleural effusion

D. Night sweats H. Atrial fibrillation

Part III Handouts v.1.0 Page 82 of 142


Answer the following three (3) questions based on the patient
information provided on the previous page.

01 . Select the three (3) most common extra-pulmonary sites of involvement. (CHOOSE 3)

A. Kidney E. Penis

B. Vertebrae F. skin

C. Lymph nodes G. Small intestine

D. Eye H. Brain

02 . Which of the following are risk factors for the condition presented. (CHOOSE 3)

A. Bronchial asthma E. HIV infection

B. Alcoholism F. Hyperthyroidism

C. Rib fractures G. Diabetes mellitus

D. Vegetarianism H. Congestive heart failure

03 . What additional clinical findings can be expected to appear? (CHOOSE 3)

A. Rhinitis E. Diarrhea

B. Constipation F. Anorexia

C. Hearing loss G. Pleural effusion

D. Night sweats H. Atrial fibrillation

Part III Handouts v.1.0 Page 83 of 142


PATIENT INFORMATION

SEX: F AGE: 70

SUBJECTIVE FINDINGS:

CHIEF COMPLAINT: Paresthesias in first three digits of both hands

MECHANISM OF INJURY: Insidious

QUALITY OF PAIN: Tingling; pins and needles

SEVERITY OF PAIN: Mild

AGGRAVATING FACTORS: Sleep

ASSOCIATED SYMPTOMS: Fatigue

RELIEVING FACTORS: Shaking out hands

PAST MEDICAL HISTORY: Hodgkin’s disease, age 37

FAMILY HISTORY: Addison disease, mother

OBJECTIVE FINDINGS:

VITALS:
LEFT BP: 122/98 mmHg
RESP: 14 cpm
PULSE: 58 bpm
TEMP: 96.7 F

RANGE OF MOTION:
Within normal limits.

ORTHOPEDIC TESTING:
Positive Tinel test at wrists.

LABORATORY STUDIES:
Elevated TSH.

OTHER FINDINGS:
Delayed relaxation phase of deep tendon
reflexes.

Part III Handouts v.1.0 Page 84 of 142


Answer the following three (3) questions based on the patient
information provided on the previous page.

01 . Which three (3) additional clinical manifestations would you expect? (CHOOSE 3)

A. Loss of lateral eyebrows E. Heat intolerance

B. Cold intolerance F. Diarrhea

C. Hyperhidrosis G. Anxiety

D. Hypercalcemia H. Coarse, sparse, dry hair

02 . Choose three (3) additional ortho/neuro tests that should be performed. (CHOOSE 3)

A. Bunnel-Littler test E. Biceps reflex

B. Swallowing test F. Upper limb tension test

C. Phalen test G. Dermatome testing

D. Cozen test H. Neer test

03 . Which three (3) diagnoses does the patient have? (CHOOSE 3)

A. Recurrent Hodgkin disease E. Carpal tunnel syndrome

B. Cardiomyopathy F. Cervical disc herniation

C. Primary Hypothyroidism G. Cubital tunnel syndrome

D. Primary Hyperthyroidism H. Hypertension

Part III Handouts v.1.0 Page 85 of 142


Answer the following three (3) questions based on the patient
information provided on the previous page.

01 . Which three (3) additional clinical manifestations would you expect? (CHOOSE 3)

A. Loss of lateral eyebrows E. Heat intolerance

B. Cold intolerance F. Diarrhea

C. Hyperhidrosis G. Anxiety

D. Hypercalcemia H. Coarse, sparse, dry hair

02 . Choose three (3) additional ortho/neuro tests that should be performed. (CHOOSE 3)

A. Bunnel-Littler test E. Biceps reflex

B. Swallowing test F. Upper limb tension test

C. Phalen test G. Dermatome testing

D. Cozen test H. Neer test

03 . Which three (3) diagnoses does the patient have? (CHOOSE 3)

A. Recurrent Hodgkin disease E. Carpal tunnel syndrome

B. Cardiomyopathy F. Cervical disc herniation

C. Primary Hypothyroidism G. Cubital tunnel syndrome

D. Primary Hyperthyroidism H. Hypertension

Part III Handouts v.1.0 Page 86 of 142


PATIENT INFORMATION

SEX: M AGE: 4

SUBJECTIVE FINDINGS:

CHIEF COMPLAINT: Left anteromedial knee pain

MECHANISM OF INJURY: Gradual

QUALITY OF PAIN: Achy

SEVERITY OF PAIN: moderate

AGGRAVATING FACTORS: activity, jumping, running

ASSOCIATED SYMPTOMS: left groin tenderness

RELIEVING FACTORS: rest

PAST MEDICAL HISTORY: unremarkable

FAMILY HISTORY: non-contributory

OBJECTIVE FINDINGS:

VITALS:
LEFT BP: 100/60 mmHg
RESP: 24 cpm
PULSE: 108 bpm
TEMP: 99.7ºF

RANGE OF MOTION:
Left hip ROM is reduced in abduction and
internal rotation

ORTHOPEDIC TESTING:
Negative Clarke test

LABORATORY STUDIES:
None performed

OTHER FINDINGS:
Child limps

Part III Handouts v.1.0 Page 87 of 142


Answer the following three (3) questions based on the patient
information provided on the previous page.

01 . What additional tests should be performed on this patient? (CHOOSE 3)

A. CBC E. Valsalva maneuver

B. Patrick FABERE F. Opthalmoscopic exam

C. Bone scan G. Apley’s distraction and compression

D. Lumbar spine x-rays H. Goldthwaite test

02 . What are correct regarding the etiology of this condition? (CHOOSE 3)

A. Obesity E. Due to infection

B. Idiopathic F. More common in males

C. Most common onset in puberty G. More common in females

D. Osteonecrosis H. Due to knee trauma

03 . What are the radiology findings that you will see in this condition? (CHOOSE 3)

A. Mushroom cap sign E. Ice cap appearance

B. Increased medial joint space F. Decreased joint space

C. Smaller femoral head G. Cat bite lesion

D. Compression H. Putti’s triad

Part III Handouts v.1.0 Page 88 of 142


Answer the following three (3) questions based on the patient
information provided on the previous page.

01 . What additional tests should be performed on this patient? (CHOOSE 3)

A. CBC E. Valsalva maneuver

B. Patrick FABERE F. Opthalmoscopic exam

C. Bone scan G. Apley’s distraction and compression

D. Lumbar spine x-rays H. Goldthwaite test

02 . What are correct regarding the etiology of this condition? (CHOOSE 3)

A. Obesity E. Due to infection

B. Idiopathic F. More common in males

C. Most common onset in puberty G. More common in females

D. Osteonecrosis H. Due to knee trauma

03 . What are the radiology findings that you will see in this condition? (CHOOSE 3)

A. Mushroom cap sign E. Ice cap appearance

B. Increased medial joint space F. Decreased joint space

C. Smaller femoral head G. Cat bite lesion

D. Compression H. Putti’s triad

Part III Handouts v.1.0 Page 89 of 142


PATIENT INFORMATION

SEX: F AGE: 35

SUBJECTIVE FINDINGS:

CHIEF COMPLAINT: Low back pain

MECHANISM OF INJURY: Unexplained acute exacerbation of existing low back pain

QUALITY OF PAIN: Sharp

SEVERITY OF PAIN: 9/10

AGGRAVATING FACTORS: Lumbar extension

ASSOCIATED SYMPTOMS: Nausea and vomiting; urinary frequency

RELIEVING FACTORS: slight relief with lumbar flexion

PAST MEDICAL HISTORY: undiagnosed low back pain following weight lifting injury in school

FAMILY HISTORY: Mother: breast cancer

OBJECTIVE FINDINGS:

VITALS:
LEFT BP: 120/80 mmHg
RESP: 20 cpm
PULSE: 90 bpm
TEMP: 100.7ºF

RANGE OF MOTION:
Decreased lumbar flexion

ORTHOPEDIC TESTING:
Ely’s heel-to-buttock produces low back pain

LABORATORY STUDIES:
Microscopic hematuria

OTHER FINDINGS:
Costovertebral angle tenderness

Part III Handouts v.1.0 Page 90 of 142


Answer the following three (3) questions based on the patient
information provided on the previous page.

01 . Select the most appropriate differential diagnoses. (CHOOSE 3)

A. Acute low back strain E. Perforated peptic ulcer

B. Facet syndrome F. Acute lumbar disc prolapse

C. Ruptured abdominal aortic aneurysm G. Nephrolithiasis

D. Spondylolisthesis H. PID

02 . What is the most appropriate initial case management? (CHOOSE 3)

A. Effleurage E. Lumbar McKenzie extension exercises

B. Vigorous oral hydration F. Referral to urologist

C. Referral to vascular surgeon G. Recommend NSAIDs for pain

D. Take lumbar radiographs H. Referral to internist

03 . What is the most appropriate long term case management? (CHOOSE 3)

A. Core stabilization exercises E. Diet modification

B. Avoid manipulation in extension F. Regular preventative Graston to low back

C. Russian stim to erector spinae G. Weekly supportive spinal manipulation

D. Use condoms to prevent recurrences H. Monitor blood pressure

Part III Handouts v.1.0 Page 91 of 142


Answer the following three (3) questions based on the patient
information provided on the previous page.

01 . Select the most appropriate differential diagnoses. (CHOOSE 3)

A. Acute low back strain E. Perforated peptic ulcer

B. Facet syndrome F. Acute lumbar disc prolapse

C. Ruptured abdominal aortic aneurysm G. Nephrolithiasis

D. Spondylolisthesis H. PID

02 . What is the most appropriate initial case management? (CHOOSE 3)

A. Effleurage E. Lumbar McKenzie extension exercises

B. Vigorous oral hydration F. Referral to urologist

C. Referral to vascular surgeon G. Recommend NSAIDs for pain

D. Take lumbar radiographs H. Referral to internist

03 . What is the most appropriate long term case management? (CHOOSE 3)

A. Core stabilization exercises E. Diet modification

B. Avoid manipulation in extension F. Regular preventative Graston to low back

C. Russian stim to erector spinae G. Weekly supportive spinal manipulation

D. Use condoms to prevent recurrences H. Monitor blood pressure

Part III Handouts v.1.0 Page 92 of 142


Part III EMCQ fact sheets CCCLA National Board Review

CONDITION: Abdominal Aortic Aneurysm

Relevant anatomy:
! Most AAAs occur between the renal arteries and the aortic bifurcation (L1-L4).

Risk factors and causes:


! Smoking, hypertension, male, increasing age.

Clinical findings: ! Most are asymptomatic until rupture. May have steady, deep,
boring lumbosacral pain. Palpable enlarged aneurysm may be felt. A bruit may be
heard. At rupture: Severe abdominal and/or LBP, hypotension, tachycardia,
lightheadedness, syncope, leg pain and weakness.

Ortho/neuro tests positive:


! None.

Lab tests positive:


! None.

Radiology findings:
! DUS, CT.

Red flags for additional assessment/referral:


! Signs of rupture (see above).

Complications or sequelae:
! Death, renal failure, MI, CVA.

Outcome assessments:
!

Prognosis/expected progression:
! Without treatment: fatal. With treatment, fair chance of survival.

Initial case management:


! Call 911.

Long-term case management:


! Control BP, quit smoking.

Home-instruction/work ergonomics:
! Proper diet, low in cholesterol: DASH diet.

1
Part III Handouts v.1.0 Page 93 of 142
Part III EMCQ fact sheets CCCLA National Board Review

CONDITION: Adhesive Capsulitis

Relevant anatomy:
! The glenohumeral joint capsule develops adhesions/fibrosis.

Risk factors and causes:


! Trauma, immobilization of shoulder, DM, RSDS.

Clinical findings:
! Initially, pain with progressive restriction, eventually becoming less painful with
typical capsular pattern of restriction greatest in abduction, external rotation, and flexion.

Ortho/neuro tests positive:


! Apley scratch test, Mazion shoulder rock maneuver

Lab tests positive:


! Fasting blood glucose to R/O DM if necessary.

Radiology findings:
!

Red flags for additional assessment/referral:


!

Complications or sequelae:
!

Outcome assessments:
! ROM, upper extremity functional scale

Prognosis/expected progression:
! Good with treatment; takes months to recover

Initial case management:


! Passive mobilization, stretching, USD

Long-term case management:


! Continued stretching; strengthening of atrophied muscles

Home-instruction/work ergonomics:
! Codmanʼs exercises, wall-walking exercises

2
Part III Handouts v.1.0 Page 94 of 142
Part III EMCQ fact sheets CCCLA National Board Review

CONDITION: Ankylosing spondylitis

Relevant anatomy:
! Axial joints, typically starting with SI joints, become inflamed, then ankylose.

Risk factors and causes:


! Male, caucasian, young adults.

Clinical findings:
! Low back pain and stiffness. Stiffness worst in morning (>1 hr duration), and after
prolonged inactivity. Back pain worse at night. May have low grade fever, anorexia, weight loss,
fatigue. Decreased spinal ROM, initially in low back, but progressive cephalad as ankylosis
advances up the spine. May cause decreased chest expansion (<2” for males or <1½” for
females). May develop uveitis. Peripheral joints may be involved as well.

Ortho/neuro tests positive:


! Iliac compression, gapping, Elyʼs heel to buttock, Gillet.

Lab tests positive:


! HLA-B27, ESR.

Radiology findings:
! Initially widening of SI joints eventually followed by ankylosis. Vertebral column can
demonstrate any number of findings, including marginal syndesmophytes, dagger sign, trolley
track sign, shiny corner sign, Romanus lesion, bamboo spine, barrel vertebra, squared off
vertebra.

Red flags for additional assessment/referral:


! Acute back pain following trauma--high risk of fracture of ankylosed segments. Be ware
of ADI instability.

Complications or sequelae:
!
Outcome assessments:
! Monitor spinal ROM; disability scores (ie Oswestry, Rowland-Morris).

Prognosis/expected progression:
! Not curable. Can slow or halt progression.

Initial case management:


! Refer to rheumatologist. Anti-inflammatory PT; gentle mobilization.

Long-term case management:


! Exercises to maintain posture and strength, continued mobilization.

Home-instruction/work ergonomics:
! Perform home exercises; emphasize fish, fruits and vegetables.

3
Part III Handouts v.1.0 Page 95 of 142
Part III EMCQ fact sheets CCCLA National Board Review

CONDITION: Cervical spondylotic myelopathy

Relevant anatomy:
! Cervical DJD, disc herniation, or tumor is compressing spinal cord (and most
likely also cervical nerve roots).

Risk factors and causes:


! Increasing age, prior trauma.

Clinical findings:
! Neck pain with possible arm radiation with numbness, weakness and
hyporeflexia in arm(s), along with long tract signs in the lower extremities (Babinski,
hyperreflexia, spasticity, loss of vibration and proprioception--abnormal gait--, and
abnormal pain and temp sensation).

Ortho/neuro tests positive:


! See above for neuro; orthos: cervical compression and distraction positive;
possibly Soto-Hall.

Lab tests positive:


!

Radiology findings:
! Cervical DJD; MRI or CT may show disc herniation or tumor.

Red flags for additional assessment/referral:


! Bowel or bladder dysfunction, suggesting severe cord compression.

Complications or sequelae:
! Permanent loss of neurological function in lower extremities.

Outcome assessments:
! Monitor neurologic status.

Prognosis/expected progression:
! Fair to good with appropriate management.

Initial case management:


! Refer to neurosurgeon.

Long-term case management:


!

Home-instruction/work ergonomics:
!

4
Part III Handouts v.1.0 Page 96 of 142
Part III EMCQ fact sheets CCCLA National Board Review

CONDITION: Cluster headache

Relevant anatomy:
!

Risk factors and causes:


! Men who smoke. 20-40 years old. Alcohol triggers HA.

Clinical findings:
! HA usually occurs at same time each day, often in the middle of the night.
Excruciating sudden pain, usually over eye with nasal congestion, lacrimation, facial
flushing, Hornerʼs syndrome. The patient is agitated and restless. Resolves in 30-60
min. Recurs daily for a while, then abates. Diagnosis is based on Sx characteristics.

Ortho/neuro tests positive:


!

Lab tests positive:


!

Radiology findings:
!

Red flags for additional assessment/referral:


!

Complications or sequelae:
!

Outcome assessments:
!

Prognosis/expected progression:
! Relapsing/remitting.

Initial case management:


! Melatonin. Referral.

Long-term case management:


!

Home-instruction/work ergonomics:
! Avoid alcohol; stop smoking.

5
Part III Handouts v.1.0 Page 97 of 142
Part III EMCQ fact sheets CCCLA National Board Review

CONDITION: COPD

Relevant anatomy:
! Consists of chronic obstructive bronchitis and emphysema. Chronic bronchitis becomes chronic
obstructive bronchitis when spirometric evidence of airflow obstruction develops. The bronchial walls are
narrowed, swollen and inflammed, and contain sputum. Emphysema is destruction of lung parenchyma
leading to loss of elastic recoil and alveolar septa.

Risk factors and causes:


! Most cases are due to cigarette smoking.

Clinical findings:
! Productive cough, progressive dyspnea, recurrent acute lung infections, prolonged expiration with
pursed lip breathing, barrel chest, wheezing and rhonchi.

Ortho/neuro tests positive:


! N/A

Lab tests positive:


! Spirometry: reduced FEV1 and FVC (forced expiratory volume in 1 second and forced vital
capacity)

Radiology findings:
! Flattening of diaphragm, narrow heart, rapid tapering of hilar vessel, peribronchial cuffing, and
widening of retrosternal space.

Red flags for additional assessment/referral:


! Increased work of breathing, esp with cyanosis, somnolence, low O2 saturation on pulse
oximetry, chest x-ray indicative of pneumonia or pneumothorax.

Complications or sequelae:
! Pneumonia, spontaneous pneumothorax, acute exacerbation of bronchitis, cor pulmonale (right
ventricular enlargement and failure).

Outcome assessments:
! Periodic spirometry.

Prognosis/expected progression:
! Mortality increases with decreasing FEV1. Death can also occur with acute pneumonia or cardiac
disease. Progression of COPD will continue if patient continues to smoke.

Initial case management:


! Referral to pulmonologist.

Long-term case management:


! Smoking cessation; medical.

Home-instruction/work ergonomics:
! Encourage smoking cessation.

6
Part III Handouts v.1.0 Page 98 of 142
Part III EMCQ fact sheets CCCLA National Board Review

CONDITION: Cushingʼs

Relevant anatomy:
! Excessive adrenal cortisol production.

Risk factors and causes:


! There are three organic causes of Cushingʼs: most cases are due to a pituitary tumor produces
excess ACTH. This type is called Cushingʼs disease. A second cause is an adrenal tumor producing
excess cortisol. The third cause is ectopic ACTH production, most often by lung cancers. These second
two are called Cushingʼs syndrome.

Clinical findings:
! Moon facies with a plethoric appearance, truncal obesity with proximal muscle atrophy and
weakness (potato on toothpicks appearance), suprclavicular and cervicodorsal fat pads (buffalo hump),
thin atrophic skin with purple striae, easy bruising and poor wound healing, hypertension, mental
disturbances, menstrual irregularities, frontal balding, and hirsutism.

Ortho/neuro tests positive:


!

Lab tests positive:


! Hyperglycemia. In Cushingʼs disease and ectopic ACTH production, ACTH and cortisol are
elevated. In Cushingʼs syndrome due to adrenal tumor, ACTH is low and cortisol is elevated.

Radiology findings:
! Osteoporosis. Possible renal calculi.

Red flags for additional assessment/referral:


!

Complications or sequelae:
!

Outcome assessments:
!

Prognosis/expected progression:
! Good to poor.

Initial case management:


! Referral.

Long-term case management:


! Medical.

Home-instruction/work ergonomics:
!

7
Part III Handouts v.1.0 Page 99 of 142
Part III EMCQ fact sheets CCCLA National Board Review

CONDITION: Endometriosis

Relevant anatomy:
! Functioning endometrial tissue is implanted outside the uterine cavity. In most cases, it is
confined to the peritoneal cavity, especially the ovaries, broad ligaments, posterior cul-de-sac, and
uterosacral ligaments. Less commonly, it may implant on the serosal surfaces of the intestines, ureters,
bladder, vagina, cervix, pleura and pericardium. Bleeding from the tissues causes inflammation and
subsequent adhesions.

Risk factors and causes:


! Ectopic tissue results from retrograde flow of menstrual tissue through the fallopian tubes.

Clinical findings:
! Common: pelvic pain, especially during menses, pelvic mass, altered menses, infertility,
dyspareunia. The onset of dysmenorrhea after years of painfree menses is a particularly important
diagnostic clue. Other possible symptoms depend on sites of implantation: abdominal bloating, painful
defecation, painful urination, rectal bleeding, etc.

Ortho/neuro tests positive:


!

Lab tests positive:


! Definitive Dx is by laparoscopic biopsy.

Radiology findings:
! CT/MRI may show lesions.

Red flags for additional assessment/referral:


!

Complications or sequelae:
!

Outcome assessments:
!

Prognosis/expected progression:
! Good to poor, depending on severity.

Initial case management:


! Referral.

Long-term case management:


! Medical.

Home-instruction/work ergonomics:
!

8
Part III Handouts v.1.0 Page 100 of 142
Part III EMCQ fact sheets CCCLA National Board Review

CONDITION: Gonococcal arthritis

Relevant anatomy:
! Most often affects small joints of hands, wrists, elbows, knees, and ankles.

Risk factors and causes:


! Unprotected sex with infected individual. Usually adolescents or young adults.

Clinical findings:
! Fever, chills, skin rash (pustules/papules), migratory tenosynovitis and arthritis
which settles in one or a few joints. May or may not have genitourinary symptoms.

Ortho/neuro tests positive:


!

Lab tests positive:


! Positive genitourinary culture, positive synovianalysis, increased WBC count,
ESR, blood cultures.

Radiology findings:
! Soft tissue swelling, synovial effusion, eventually joint destruction may occur).

Red flags for additional assessment/referral:


!

Complications or sequelae:
! Permanent joint destruction (uncommon).

Outcome assessments:
!

Prognosis/expected progression:
! Good with appropriate antibiotics.

Initial case management:


! Refer.

Long-term case management:


!

Home-instruction/work ergonomics:
! Counsel patient on sexual protection.

9
Part III Handouts v.1.0 Page 101 of 142
Part III EMCQ fact sheets CCCLA National Board Review

CONDITION: Hodgkinʼs lymphoma

Relevant anatomy:
! Malignant proliferation of lymphoreticular system.

Risk factors and causes:


! Males slightly more common than females, 15-40 yr old.

Clinical findings:
! Painless cervical lymphadenopathy, but pain may occur immediately after alcohol
consumption. Pruritus may occur. Fever, night sweats, weight loss. Splenomegaly.
(Fever may be a Pel-Ebstein fever: days of high fever alternating with days of low or
normal temperature).

Ortho/neuro tests positive:


!

Lab tests positive:


! Lymph node biopsy: Reed-Sternberg cells. Anemia, lymphocytopenia,
eosinophilia, increased alkaline phosphatase and ESR.

Radiology findings:
! Ivory vertebra. Mediastinal lymphadenopathy on CXR.

Red flags for additional assessment/referral:


!

Complications or sequelae:
! Most complications are those of treatment, however, the disease can be fatal.

Outcome assessments:
!

Prognosis/expected progression:
! Cure in >75%.

Initial case management:


! Refer.

Long-term case management:


!

Home-instruction/work ergonomics:
!

10
Part III Handouts v.1.0 Page 102 of 142
Part III EMCQ fact sheets CCCLA National Board Review

CONDITION: Hypothyroidism (myxedema)

Relevant anatomy:
! Most cases are primary hypothyroidism, however, 10% are secondary. More
common in women.

Risk factors and causes:


! Primary hypothyroidism is almost always due to Hashimotoʼs thyroiditis
(autoimmune); in the US, iodine deficency is no longer a common cause; secondary is
due to hypopituitarism.

Clinical findings:
! Cold intolerance, constipation, forgetfulness, slow mentation, modest weight
gain, carpal tunnel syndrome, amenorrhea, hoarse voice, slow speech, facial puffiness,
loss of lateral eyebrows, sparse hair, coarse, dry skin, bradycardia, enlarged tongue,
possible pleural effusion.

Ortho/neuro tests positive:


! Delayed relaxation phase of deep tendon reflexes (pathognomonic).

Lab tests positive:


! In primary hypothyroidism: Elevated TSH, low T4 and T3; in secondary: low TSH,
T4, and T3. Possible anemia on CBC.

Radiology findings:
!

Red flags for additional assessment/referral:


! Coma.

Complications or sequelae:
! Myxedema coma.

Outcome assessments:
!

Prognosis/expected progression:
! Good with thyroid hormone replacement.

Initial case management:


! Referral.

Long-term case management:


! Medical.

Home-instruction/work ergonomics:

11
Part III Handouts v.1.0 Page 103 of 142
Part III EMCQ fact sheets CCCLA National Board Review

CONDITION: Infectious mononucleosis

Relevant anatomy:
! Involves cervical lymph nodes, upper respiratory tract, and spleen.

Risk factors and causes:


! Spread of EBV by kissing.

Clinical findings:
! Fatigue, lasting days to weeks, followed by fever, pharyngitis, and
lymphadenopathy. Fever peaks in afternoon or early evening. Splenomegaly occurs in
50% of cases.

Ortho/neuro tests positive:


!

Lab tests positive:


! CBC (lymphocytosis) and heterophil antibody test, (Mono-spot). Possible
elevated liver enzymes.

Radiology findings:
!

Red flags for additional assessment/referral:


! Signs of complications (see below).

Complications or sequelae:
! Meningitis, encephalitis, Guillian-Barre syndrome, cranial nerve palsies, splenic
rupture. EBV increases risk of Burkittʼs lymphoma.

Outcome assessments:
!

Prognosis/expected progression:
! Usually self-limited.

Initial case management:


! Supportive.

Long-term case management:


!

Home-instruction/work ergonomics:
! Avoid heavy lifting and contact sports to prevent splenic rupture.

12
Part III Handouts v.1.0 Page 104 of 142
Part III EMCQ fact sheets CCCLA National Board Review

CONDITION: Infective endocarditis

Relevant anatomy:
! Infection of the endocardial layer and valves of the heart.

Risk factors and causes:


! IV drug abuse, valvular disease, prosthetic valves, immunocompromise are
common risk factors. Staph aureus and strep viridans are common causes, the former
from IV drug use, the latter from dental work.

Clinical findings:
! Fever and a murmur. Also can experience fatigue, dyspnea, and bacterial
embolic phenomena: stroke, splinter hemorrhages in nailbeds, Oslerʼs nodes (painful
nodules on finger pads and toe pads), Janeway lesions (painless reddish-brown spots
on palms and soles), splenic enlargement and pain (abscesses), petechial hemorrhages
in conjunctivae, Roth spots (retinal).

Ortho/neuro tests positive:


! N/A

Lab tests positive:


! Positive blood cultures, increased WBC count, increased ESR.

Radiology findings:
!

Red flags for additional assessment/referral:


! LUQ pain, RUQ pain (splenic or hepatic abscess), stroke, signs of CHF, MI.

Complications or sequelae:
! Stroke and other organ ischemic necrosis due to emboli.

Outcome assessments:
! Negative blood cultures.

Prognosis/expected progression:
! Good with aggressive antibiotic treatment.

Initial case management:


! Referral.

Long-term case management:


! Medical.

Home-instruction/work ergonomics:

13
Part III Handouts v.1.0 Page 105 of 142
Part III EMCQ fact sheets CCCLA National Board Review

! CONDITION: Isthmic spondylolisthesis

Relevant anatomy:
! Slippage anteriorly of a vertebra. Most commonly for isthmic spondys is L5, while
L4 is most commonly involved in degererative spondy. Isthmic is most common type of
spondylolisthesis.

Risk factors and causes:


! Usually occurs in adolescence. Often due to minimal trauma with disruption of
the pars (spondylolysis), and subsequent anterior slippage (spondylolisthesis).

Clinical findings:
! Low back pain, worse in extension, or when straightening from a stooped
posture.

Ortho/neuro tests positive:


! Pheasant, Stork standing

Lab tests positive:


!

Radiology findings:
! Break in pars with anterior slippage of vertebra. Graded via Meyerding, and
Ullmanʼs line.

Red flags for additional assessment/referral:


! Cauda equina syndrome

Complications or sequelae:
! Instability, neurological compromise

Outcome assessments:
! Oswestry; monitor degree of slippage.

Prognosis/expected progression:
! Fair to good with treatment.

Initial case management:


! Flexion-distraction, supine sacral pull spondylo adjustment

Long-term case management:


! Rehab for core stability

Home-instruction/work ergonomics:
!

14
Part III Handouts v.1.0 Page 106 of 142
Part III EMCQ fact sheets CCCLA National Board Review

CONDITION: Legg-Calve-Perthesʼ disease

Relevant anatomy:
! Affects the femoral head in children.

Risk factors and causes:


! Boys > girls; 4-10 yrs old. Idiopathic aseptic necrosis of femur head.

Clinical findings:
! Hip and/or anteromedial knee pain, gait disturbance of gradual onset and slow
progression. Worse with activity and weight bearing. Abduction and internal rotation of
hip is most significantly reduced.

Ortho/neuro tests positive:


! Patrick FABERE

Lab tests positive:


!

Radiology findings:
! Flattening and opacification of femoral head. Diagnosis is confirmed with bone
scan or MRI.

Red flags for additional assessment/referral:


!

Complications or sequelae:
! Residual distortion of femoral head, and subsequent development of DJD.

Outcome assessments:
!

Prognosis/expected progression:
! Without treatment, self-resolving in 2-3 years. With treatment, sequelae are less
severe.

Initial case management:


! Prolonged bed rest, referral.

Long-term case management:


! Mobile traction, slings and splints, abduction plaster casts.

Home-instruction/work ergonomics:
!

15
Part III Handouts v.1.0 Page 107 of 142
Part III EMCQ fact sheets CCCLA National Board Review

CONDITION: Lumbar disc herniation

Relevant anatomy:
! Protrusion of a lumbar disc, most commonly L4 or L5. L4 disc (between L4 and L5)
affects the L5 nerve root. The L5 disc affects the S1 nerve root.

Risk factors and causes:


! Lifting, flexion with twisting, sedentary lifestyle, truck driver, prior back trauma, smoking.

Clinical findings:
! Acute LBP, accompanied by sciatic leg pain, often to the foot, aggravated by sitting and
flexion, less severe with standing, relieved by lying down. Dejerineʼs triad. May have loss of
neurologic function in affected nerve root (paresthesia or numbness, hyporeflexia, weakness).

Ortho/neuro tests positive:


! SLR, Braggard, WLR and Fajersztajn if medial disc, Valsalva, Kemp, Turyn. For L5
nerve root: weak great toe extensor or heel walk, or tibialis posterior, and sensory over dorsum
of foot and anterolateral lower leg. For S1 nerve root: weak toe walk or peroneii, Achillesʼ reflex,
and sensory over lateral/plantar foot.

Lab tests positive:


!

Radiology findings:
! MRI, or CT evidence of disc herniation.

Red flags for additional assessment/referral:


! Cauda equina syndrome (perineal numbness with loss of bowel/bladder function).

Complications or sequelae:
! Permanent loss of neurologic function if cauda equina is not treated promptly.

Outcome assessments:
! Monitor neurologic status, orthopedic tests; outcome measures (Oswestry, Rowland-
Morris, or Bournemouth).

Prognosis/expected progression:
! Good with proper treatment.

Initial case management:


! McKenzie protocol, flexion-distraction, analgesic and anti-inflammatory modalities. Refer
immediately if cauda equina syndrome is present. Refer to spine surgeon if patientʼs condition
deteriorates or fails to improve.

Long-term case management:


! Ergonomic assessment and education, back rehab exercises.

Home-instruction/work ergonomics:
! Avoid sitting, flexion, lifting, instruct in icing and home exercises.

16
Part III Handouts v.1.0 Page 108 of 142
Part III EMCQ fact sheets CCCLA National Board Review

CONDITION: March fracture (metatarsal stress fracture)

Relevant anatomy:
! Metatarsal stress fracture, most commonly the second metatarsal.

Risk factors and causes:


! Poorly conditioned runners or individuals who suddenly begin long distance
walking, or begin carrying heavy packs (soldiers). Risk factors include pes cavus,
osteoporosis, female triathletes.

Clinical findings:
! Forefoot pain that occurs after long workout which lessens shortly after activity is
stopped. With subsequent exercise, onset of pain is progressively earlier. Eventually
pain is severe enough to prevent activity.

Ortho/neuro tests positive:


! Forefoot squeeze (not recommended).

Lab tests positive:


!

Radiology findings:
! Standard foot series reveals fracture.

Red flags for additional assessment/referral:


!

Complications or sequelae:
!

Outcome assessments:
!

Prognosis/expected progression:
! Will take up to 12 weeks to heal.

Initial case management:


! Crutches, wooden shoes or supports. Casts rarely required.

Long-term case management:


!

Home-instruction/work ergonomics:
! Rest.

17
Part III Handouts v.1.0 Page 109 of 142
Part III EMCQ fact sheets CCCLA National Board Review

CONDITION: Myasthenia gravis

Relevant anatomy:
! Autoimmune attack of acetylcholine receptors at the myoneural junction.

Risk factors and causes:


! Young women and older men. Precipitating factors: infections, surgery, drugs.

Clinical findings:
! Weakness and easy fatiguability of muscles. Bulbar muscles are affected first,
espcially muscles of eye (diplopia, strabismus, ptosis). Symptoms worsen as day goes
on. Other facial muscles, masticatory muscles, swallowing muscles, and tongue are
affected. Eventually other muscles can be affected, including respiratory muscles. Little
muscle atrophy is seen, and DTRs are preserved. May have thymoma.

Ortho/neuro tests positive:


! Weakness of affected muscles.

Lab tests positive:


! Positive Tensilon test. Abnormal EMG.

Radiology findings:
!

Red flags for additional assessment/referral:


! Respiratory difficulty.

Complications or sequelae:
! Respiratory failure.

Outcome assessments:
!

Prognosis/expected progression:
! Poor.

Initial case management:


! Refer.

Long-term case management:


!

Home-instruction/work ergonomics:
!

18
Part III Handouts v.1.0 Page 110 of 142
Part III EMCQ fact sheets CCCLA National Board Review

CONDITION: Nephrolithiasis, Urolithiasis

Relevant anatomy:
! Nearly 80% of kidney stones are composed of calcium oxalate, 10% are
composed of uric acid, 2% are cystine, and the rest are struvite (magnesium ammonium
phosphate).

Risk factors and causes:


! For calcium oxalate stones, the main risk factors are hypercalciuria (hereditary),
or hyperoxaluria caused by excess ingestion of oxalate-rich foods (rhubarb, spinach,
chocolate). Uric acid stones are formed in patients with hyperuricemia, gout, and
excessive consumption of high-purine foods.

Clinical findings:
! Excruciating back pain with radiation across abdomen into groin, nausea and
vomiting, and hematuria. Urinary frequency and urgency are common.

Ortho/neuro tests positive:


! Murphyʼ punch produces costovertebral angle tenderness.

Lab tests positive:


! Hematuria.

Radiology findings:
! Calcium containing stones can be visible on plain film. Noncontrast spiral CT is
the diagnostic imaging modality of choice.

Red flags for additional assessment/referral:


! High fever, and shaking chills, indicating infection proximal to stone.

Complications or sequelae:
! Kidney infection and sepsis.

Outcome assessments:
!

Prognosis/expected progression:
! Good with proper treatment.

Initial case management:


! Referral.

Long-term case management:


! Diet modification to prevent recurrence.

Home-instruction/work ergonomics:

19
Part III Handouts v.1.0 Page 111 of 142
Part III EMCQ fact sheets CCCLA National Board Review

CONDITION: Osteoporosis

Relevant anatomy:
! Cortical and trabecular (cancellous) bone loss occurs. Cortices becomes thinned and trabeculae are
decreased in number.

Risk factors and causes:


! 95% is primary, 5% is secondary. There are 3 types of primary osteoporosis: 1. Idiopathic (uncommon,
occurs in children and young adults), 2. Post-menopausal (51-75 years of age), 3. Involutional or senile (due to
decreased osteoblast activity associated with aging--seen after 60 years of age). Primary osteoporosis is more
common in women. Risk factors include: smoking, excessive caffeine and alcohol consumption, and sedentary
lifestyles. Recent evidence suggests that life-long vitamin D deficiency plays an important role. Secondary
osteoporosis is due to an underlying disease.

Clinical findings:
! None, until fracture occurs, at which time bone pain will be present. Vertebral compression fractures below
T6 are common, as are fractures of the femoral neck. Multiple thoracic compression fractures can lead to a
dowagerʼs hump.

Ortho/neuro tests positive:


! Integrity of the nervous system should be assessed in cases of spinal fractures to rule out nerve root or
spinal cord compression. Numbness and weakness indicate such lesions. Hyperreflexia and Babinski signs indicate
spinal cord compression.

Lab tests positive:


! Serum calcium and phosphorus will be normal in primary osteoporosis. Many patients with osteoporosis will
have low 25-OH vitamin D levels, however.

Radiology findings:
! Plain radiographs demonstrate osteopenia with decreased bone density and pencil thin cortices, but not until
at least 30% of bone mass has been lost. Fractures, if present, will also be seen. The gold standard for measuring
bone density is the DEXA (dual-energy x-ray absorptiometry). Screening is recommended for women over 65

Red flags for additional assessment/referral:


! Signs of underlying disease, suggesting secondary osteoporosis. Common causes of secondary
osteoporosis include chronic renal failure, COPD, corticosteroid use, Cushingʼs, hyperparathyroidism, malabsorption
syndrome, RA, and malignancy.

Complications or sequelae:
! Fracture.

Outcome assessments:
! Track DEXA scores.

Prognosis/expected progression:
! The goal of treatment is to preserve bone mass. The rate of bone loss can be slowed with aggressive
treatment.

Initial case management:


! Calcium (1000 mg per day) and vitamin D (1000 IU per day). Referral for bisphosphonate prescription, which
significantly can increase bone mass. If fracture has occurred, appropriate treatment is required.

Long-term case management:


! Continued supplement and medication. Rehabilitation post-fracture may be required.

Home-instruction/work ergonomics:
! Regular exercise (if no fracture) to encourage bone formation. If fresh compression fracture, avoid bending
and lifting, use back brace.

20
Part III Handouts v.1.0 Page 112 of 142
Part III EMCQ fact sheets CCCLA National Board Review

CONDITION: Osteosarcoma

Relevant anatomy:
! Most commonly involves the metaphysis, most common at the knee.

Risk factors and causes:


! 10-20 year olds.

Clinical findings:
! Bone pain, often deep, worse at night, unrelenting, constant.

Ortho/neuro tests positive:


!

Lab tests positive:


! Dx requires biopsy.

Radiology findings:
! Sunburst, spiculated periosteal appearance, Codmanʼs triangle, soft-tissue mass.
Do chest x-ray since lung met is common.

Red flags for additional assessment/referral:


! Cough.

Complications or sequelae:
! Lung metastasis or other distant mets.

Outcome assessments:
!

Prognosis/expected progression:
! Poor.

Initial case management:


! Refer to oncologist.

Long-term case management:


!

Home-instruction/work ergonomics:
!

21
Part III Handouts v.1.0 Page 113 of 142
Part III EMCQ fact sheets CCCLA National Board Review

CONDITION: Piriformis syndrome

Relevant anatomy:
! Spasm of piriformis muscle, often entrapping sciatic nerve.

Risk factors and causes:


! Overuse, SIJ disorders, trochanteric disease, fibrous adhesions.

Clinical findings:
! Sciatica beginning in the buttock with positive SLR aggravated with leg in internal
rotation compared to external rotation. Piriformis taut and tender. Low back pain usually
absent.

Ortho/neuro tests positive:


! May have motor or sensory deficits in the leg. Positive LLTT.

Lab tests positive:


!

Radiology findings:
! In the elderly, consider imaging to rule out spinal stenosis.

Red flags for additional assessment/referral:


!

Complications or sequelae:
!

Outcome assessments:
! Appropriate outcome measures.

Prognosis/expected progression:
! Good in most cases.

Initial case management:


! May need to treat underlying cause. Various methods of stretching muscle, e-
stim, US, CMT to SIJ if appropriate.

Long-term case management:


! Rehab may be appropriate.

Home-instruction/work ergonomics:
! Stretching exercises.

22
Part III Handouts v.1.0 Page 114 of 142
Part III EMCQ fact sheets CCCLA National Board Review

CONDITION: Community acquired pneumonia

Relevant anatomy:
! Infection of lung parenchyma.

Risk factors and causes:


! Causes include bacterial, viral, or fungal. The most common bacterial causes are: Strep
pneumoniae, Hemophilus influenzae, Chlamydia pneumoniae, and Mycoplasma pneumoniae. The most
common viral causes are: Respiratory syncytial virus (RSV), adenovirus, influenza virus, parainfluenza
virus. The most common fungal causes are Histoplasma and Coccidioides.

Clinical findings:
! Vary somewhat due to infectious agent. Generally, will produce malaise, cough, dyspnea, and
chest pain. Sputum production may be seen, especially later in course of infection. Fever, tachypnea and
tachycardia, crackles, bronchial breath sounds and egophony, and dullness to percussion may be found.
Pleural effusion may be present.

Ortho/neuro tests positive:


! N/A

Lab tests positive:


! Increased WBC count; increased PMN with bacterial, increased lymphocytes with viral, increased
monocytes with fungal. Sputum culture may be positive.

Radiology findings:
! Chest x-ray almost always demonstrates some degree of infiltrate after the first 24-48 hours.
Pleural effusion may be seen.

Red flags for additional assessment/referral:


! Presence of cyanosis, tachypnea greater than 32 breaths per minute, low O2 saturation on pulse
oximetry.

Complications or sequelae:
! Death.

Outcome assessments:
!

Prognosis/expected progression:
! Pneumococcal pneumonia accounts for 66% of all fatal cases of community acquired pneumonia
in the elderly or infants. Otherwise, prognosis tends to be good with proper treatment.

Initial case management:


! Referral; antibiotics if appropriate.

Long-term case management:


! Medical.

Home-instruction/work ergonomics:
! Rest.

23
Part III Handouts v.1.0 Page 115 of 142
Part III EMCQ fact sheets CCCLA National Board Review

CONDITION: Psoriatic arthritis

Relevant anatomy:
! Involves skin, nails, and joints, esp DIPs.

Risk factors and causes:


! AIDS, HLA-B27.

Clinical findings:
! Skin and nail involvement may precede or follow onset of arthritis. Typically is
asymmetrical involvement of DIPs of hands and feet, but can involve other joints, SI
joints and spine. Rheumatoid nodules are absent.

Ortho/neuro tests positive:


!

Lab tests positive:


! ESR and CRP increased, HLA-B27 positive in many; possible increase in uric
acid.

Radiology findings:
! In spine, asymmetric non-marginal syndesmophytes. Hand: bilateral but
asymmetrical, marginal jt erosion, fluffy periostitis (mouse ears), pencil-in-cup,
telescoping of digits leading to opera glass hand deformity; Spine: non-marginal
syndesmophytes--comma shaped, bagpipe shaped, Bywater-Dixon or floating type

Red flags for additional assessment/referral:


!

Complications or sequelae:
!

Outcome assessments:
!

Prognosis/expected progression:
! Most are controllable with medication.

Initial case management:


! Refer to rheumatologist.

Long-term case management:


!

Home-instruction/work ergonomics:

24
Part III Handouts v.1.0 Page 116 of 142
Part III EMCQ fact sheets CCCLA National Board Review

CONDITION: Rheumatoid arthritis

Relevant anatomy:
! Symmetrical inflammatory arthritis of peripheral joints.

Risk factors and causes:


! Female, 35-50 years, smoking.

Clinical findings:
! Symmetrical joint pain, swelling, stiffness lasting more than 1 hr, involving the
wrists, MCPs, and PIPs, most commonly. Symptoms worse in the morning. Low grade
fever, fatigue, anorexia. May also involve feet, elbows, ankles, and upper cervical spine.
Fingers may exhibit ulnar deviation, swan-neck and boutonniere deformities.
Rheumatoid nodules may be present.

Ortho/neuro tests positive:


!

Lab tests positive:


! Positive rheumatoid factor, elevated ESR.

Radiology findings:
! Soft tissue swelling, rat bite erosions, juxta-articular erosions, periarticular
osteopenia, uniform jt space narrowing, ulnar deviation of digits, swan neck and
boutonneire deformities, Terry Thomas sign, Lanois deformity

Red flags for additional assessment/referral:


! Remember to assess ADI for atlantodental instability.

Complications or sequelae:
! See red flags.

Outcome assessments:
! There are rheumatoid specific outcome measures.

Prognosis/expected progression:
! Unpredictable. Life-expectancy is reduced by 3-7 years.

Initial case management:


! Fish oil, anti-inflammatory PT, rest, referral to rheumatologist.

Long-term case management:


! Maintain muscle strength and mobility with exercises, as tolerated.

Home-instruction/work ergonomics:

25
Part III Handouts v.1.0 Page 117 of 142
Part III EMCQ fact sheets CCCLA National Board Review

CONDITION: SLE

Relevant anatomy:
! Multisystem, inflammatory autoimmune disorder.

Risk factors and causes:


! Female of child-bearing age, Black > White.

Clinical findings:
! Fever, polyarthritis which is typically non-destructive but may cause ulnar drift or
swan-neck deformities, malar rash, photosensitivity, oral ulcers, serositis, renal
disorders, leukopenia, headache or stroke or neuropathy or personality changes--any or
all.

Ortho/neuro tests positive:


!

Lab tests positive:


! ANA, anti-DNA, anti-SM antibodies positive, low WBC, high ESR.

Radiology findings:
! Soft-tissue swelling, vascular calcification, relative preservation of jt space, ulnar
deviation of digits, swan neck and boutonneire deformities, osteonecrosis.

Red flags for additional assessment/referral:


! Stroke, acute nephritis, sudden worsening of symptoms.

Complications or sequelae:
!

Outcome assessments:
!

Prognosis/expected progression:
! Relapsing and remitting.

Initial case management:


! Refer to rheumatologist.

Long-term case management:


!

Home-instruction/work ergonomics:
!

26
Part III Handouts v.1.0 Page 118 of 142
Part III EMCQ fact sheets CCCLA National Board Review

CONDITION: Degenerative ;umbar spinal stenosis

Relevant anatomy:
! Narrowing of the central canal and often, the lateral recesses.

Risk factors and causes:


! Degenerative. Risk of symptoms increase in congenitally small canals.

Clinical findings:
! Possible low back pain and morning stiffness lasting less than 1 hr. Unilateral,
bilateral, or alternating leg pain, radiculopathy, neurogenic claudication, possible
bowel/bladder involvement.

Ortho/neuro tests positive:


! Pheasant test, stoop test, bicycle test.

Lab tests positive:


!

Radiology findings:
! Possible degenerative spondylolisthesis, lipping and spurring, facet hypertrophy,
positive Eisensteinʼs measurement. Need CT or MRI to adequately evaluate.

Red flags for additional assessment/referral:


!

Complications or sequelae:
!

Outcome assessments:
! Oswestry or similar.

Prognosis/expected progression:
! Fair. Can be managed in many cases, not cured without surgery.

Initial case management:


! Flexion distraction, Williams flexion exercises

Long-term case management:


! Continue flexion exercises; postural correction.

Home-instruction/work ergonomics:
! Avoid prolonged extension.

27
Part III Handouts v.1.0 Page 119 of 142
Part III EMCQ fact sheets CCCLA National Board Review

CONDITION: Type 2 diabetes mellitus

Relevant anatomy:
! Beta islet cells initially secrete high levels of insulin due to insulin receptor resistance, but eventually insulin
production falls, further exacerbating the hyperglycemia.

Risk factors and causes:


! Obesity, American Indians, Hispanics, Asians.

Clinical findings:
! Polyuria, polydipsia, excessive thirst. Fatigue, weakness. Often asymptomatic. May present with signs and
symptoms of complications.

Ortho/neuro tests positive:


!
Lab tests positive:
! Elevated fasting glucose, elevated HbA1c.

Radiology findings:
!
Red flags for additional assessment/referral:
! Signs and symptoms of complications. (See below).

Complications or sequelae:
1. Diabetic retinopathy. This is the most common cause of blindness in the US. Characterized by retinal
microaneurysms, macular edema, neovascularization, cotton-wool spots, and retinal detachment.
2. Diabetic nephropathy. This is the most common cause of renal failure in the US. Characterized by systemic
hypertension, albuminuria, and nephrotic syndrome, with wide-spread edema.
3. Diabetic neuropathy. This is the most common cause of peripheral polyneuropathy in the US.
Characterized by glove and stocking paresthesia, loss of proprioception. May also have
mononeuropathies, especially of median, peroneal, femoral. May also develop autonomic neuropathy,
characterized by gastroparesis and bezoar formation, orthostatic hypotension, diarrhea, and abnormal
sweating patterns.
4. MI, CHF, CVA, PAD.
5. Foot ulcers and gangrene. DM is the most common cause of gangrene in the US.
6. Infection. Diabetics are more prone to infection than non-diabetics.
7. Dupuytrenʼs contracture, adhesive capsulitis
8. Necrobiosis lipoidica diabeticorum, acanthosis nigricans.

Outcome assessments:
! Daily blood glucose monitoring.

Prognosis/expected progression:
! Without proper treatment, prognosis is poor; with proper treatment, development of complication is reduced
and prognosis is fair.

Initial case management:


! Referral. Diet.

Long-term case management:


! Medical and dietary.

Home-instruction/work ergonomics:
! Encourage proper blood glucose monitoring, dietary adherence, and exercise.

28
Part III Handouts v.1.0 Page 120 of 142
Part III EMCQ fact sheets CCCLA National Board Review

CONDITION: Tuberculosis

Relevant anatomy:
! Mycobacterial infection most commonly affecting the lungs. In HIV negative patients, it most
typically affects the lung apex. In HIV positive patients, it may affect any portion of lungs, and may have a
miliary distribution.

Risk factors and causes:


! The most significant risk factor is HIV infection. Other risk factors include homelessness, living in
crowded living conditions, alcoholism, malnutrition, recent immigration from high-prevalence countries:
Southeast Asia, Central America, Africa.

Clinical findings:
! Progressive cough, eventually productive of purulent sputum, eventually containing blood; chest
pain, esp pleuritic, weight loss, night sweats, fever, crackles.

Ortho/neuro tests positive:


! N/A

Lab tests positive:


! Positive PPD (purified protein derivative skin test); elevated WBC count, esp monocytes; positive
sputum stain for acid-fast bacilli and culture.

Radiology findings:
! Varies. Multinodular infiltrate above or posterior to clavicle, middle or lower lung infiltrate, upper
lobe lung cavitation (classic), pleural effusion.

Red flags for additional assessment/referral:


! Additional assessment required if there is a suggestion of extra-pulmonary involvement.

Complications or sequelae:
! Complications include: renal involvement, spinal involvement (Pott disease), lymph node
involvement, or meningitis, peritonitis, and pericarditis.

Outcome assessments:
!

Prognosis/expected progression:
! Generally good with proper treatment, although MDRTB (multi-drug resistant TB) may be more
challenging to cure.

Initial case management:


! Referral.

Long-term case management:


! Medical.

Home-instruction/work ergonomics:
! Reinforce need to continue with antibiotic regimen as instructed by physician.

29
Part III Handouts v.1.0 Page 121 of 142
Part III EMCQ fact sheets CCCLA National Board Review

CONDITION: Thoracic outlet syndrome

Relevant anatomy:
! Entrapment of neurovascular bundle in thoracic outlet.

Risk factors and causes:


! Entrapping structure may be scalenus anticus, cervical rib, clavicle (postural), pec minor,
tumor of lung apex of lymph nodes.

Clinical findings:
! May have neck or shoulder pain with radiation down the arm. If primarily brachial plexus
is entrapped, will see paresthesia/numbness, possible weakness, and decreased DTRs.
Raynaudʼs may be seen. If primarily the subclavian artery and/or vein is entrapped, will see
color changes (pallor or cyanosis), cold extremity, Raynaudʼs, weak pulses, swelling in hand.
Neurologic entrapment is much more common.

Ortho/neuro tests positive:


! Adsonʼs, costoclavicular, Wrightʼs/hyperabduction, Allen

Lab tests positive:


!

Radiology findings:
! Cervical films to rule out cervical rib; CXR and apical lordotic if Pancoast tumor is
suspected.

Red flags for additional assessment/referral:


! Lymphadenopathy, history of smoking.

Complications or sequelae:
! In severe cases of vascular involvement, may see gangrene of fingertips. In neurologic
involvement, may see wasting in intrinsic muscles of hand.

Outcome assessments:
! Upper extremity functional scale.

Prognosis/expected progression:
! Poor, to fair to good, depending on cause.

Initial case management:


! Determine cause, and whether referral is required. Treat underlying musculoskeletal
fault, including stretching, myofascial release, CMT.

Long-term case management:


! Postural correction and exercises.

Home-instruction/work ergonomics:
! See above.

30
Part III Handouts v.1.0 Page 122 of 142
Part III EMCQ fact sheets CCCLA National Board Review

CONDITION: Urinary tract infection (UTI)

Relevant anatomy:
! Can involve urethra, bladder, prostate, kidney.

Risk factors and causes:


! Female gender. Diabetes mellitus, catheterization, prostate enlargement. Most common
infectious organisms: E. Coli.

Clinical findings:
! Urethritis: dysuria, urethral discharge.
! Cystitis: sudden onset of frequency, urgency, and painful voiding of small volumes of urine.
Hematuria may be present.
! Prostatitis: suprapubic or rectal pain, urethral discharge, frequency, urgency, difficulty stopping,
dribbling.
! Pyelonephritis: low back or flank pain. Urinary frequency and dysuria is seen in 33%. Chills, fever,
nausea and vomiting are typical. Murphyʼs punch reveals costovertebral angle tenderness.

Ortho/neuro tests positive:


!

Lab tests positive:


! Urinalysis: culture and sensitivity reveals organism; positive nitrite and leukocyte esterase
indicates presence of infection.

Radiology findings:
!

Red flags for additional assessment/referral:


! Signs of pyelonephritis (the most severe form of UTI).

Complications or sequelae:
!

Outcome assessments:
!

Prognosis/expected progression:
! Good.

Initial case management:


! Referral.

Long-term case management:


! Cranberry juice daily can reduce risk of recurrence.

Home-instruction/work ergonomics:
! Cranberry juice.

31
Part III Handouts v.1.0 Page 123 of 142
PART III Miscellaneous Factoids! CCCLA National Board Review

Worst-case causes of acute abdominal pain

Children:! ! Appendicitis
! ! ! Intussusception
! ! ! Reyeʼs syndrome

Young women:! Appendicitis


! ! ! Ectopic pregnancy
! ! ! PID

Young men:! ! Appendicitis


! ! ! Testicular torsion

Midlife adults:! Acute MI


! ! ! Perforated ulcer

Elderly adults:! Acute intestinal ischemia


! ! ! Necrotic bowel
! ! ! Bowel obstruction
! ! ! Ruptured abdominal aortic aneurysm

1
Part III Handouts v.1.0 Page 124 of 142
PART III Miscellaneous Factoids! CCCLA National Board Review

Features of inflammatory vs mechanical back pain


Mechanical Inflammatory
Onset acute gradual
Age at onset any age usually < 40
(years)
Effect of worse better
exercise
Effect of rest better worse
Morning + +++
stiffness
Pain radiation nerve root diffuse
irritation/ tension
signs- L4, L5, S1
Sleep +/– +++
disturbance
Tenderness, local diffuse
spasm
Scoliosis + –
Deficit in asymmetrical symmetrical
movement
range

2
Part III Handouts v.1.0 Page 125 of 142
PART III Miscellaneous Factoids! CCCLA National Board Review

RHEUMATOLOGIC FINDING CAUSES/DIAGNOSES


PAINFUL, SWOLLEN HANDS •Gout
•Pseudogout
•RA
•Scleroderma (early)
•CRPS (complex regional pain syndrome,
formerly RSD or RSDS)
ACUTE MONOARTHRITIS (PAINFUL, SWOLLEN, •Septic arthritis
WARM JOINT) •Gout
•Pseudogout
•Intraarticular fracture
•Hemarthrosis
CAUSES OF HEMARTHROSIS •Trauma
•Intraarticular fracture
•Hemophilia
•Osteonecrosis
ARTHRITIS, HIGH FEVER, AND RASH •Rubella
•Fifth disease (parvovirus B-19)
•Gonococcemia or meningococcemia
•Secondary syphilis
•Lyme disease
ARTHRITIS AND A RASH •SLE
•Dermatomyositis
•Psoriatic arthritis
•Reactive arthritis (Reiterʼs)
•Vasculitis
ACUTE ONSET OF POLYARTHRITIS AND FEVER •Vasculitis
•JRA
•Endocarditis
•Rheumatic fever
CHRONIC MONOARTHRITIS •Mechanical derangement
•Neuropathic joint
•RA
•Psoriatic arthritis
CHRONIC DISTAL SYMMETRICAL •RA
POLYARTHRITIS •SLE
•Polymyositis
•Gout or pseudogout
•Psoriatic arthritis
ARTHRITIS AND SUBCUTANEOUS NODULES •RA
•Gout (tophi)
•Sarcoidosis
•Rheumatic fever
DIP JOINT ARTHRITIS •OA (Heberdenʼs nodes type)
•Nodal gout
•Psoriatic arthritis
OLIGOARTHRITIS •Psoriatic arthritis
•Lyme disease
•Reactive arthritis
•Gout
•Pseudogout

3
Part III Handouts v.1.0 Page 126 of 142
PART III Miscellaneous Factoids! CCCLA National Board Review

SPONDYLITIS •AS*
•Reactive arthritis*
•Enteropathic arthritis*
•Psoriatic arthritis*
•RA**
*Seronegative
**Seropositive
ENTHESOPATHY •Reactive arthritis
•AS
•Psoriatic arthritis
•DISH
ARTHRITIS AND WEIGHT LOSS •Severe RA or RA with vasculitis
•Reactive arthritis
•Cancer
•Enteropathic arthritis
•HIV
ARTHRITIS AND A HEART MURMUR •Endocarditis
•AS
•Reactive arthritis
•Rheumatic fever
•RA or SLE
JOINT PAIN AND MUSCLE WEAKNESS •RA
•AS
•Polymyositis/dermatomyositis
•SLE
•Progressive systemic sclerosis
PROXIMAL JOINT PAIN AND STIFFNESS •Polymyalgia rheumatica
•Osteomalacia
•Multiple myeloma
•Metastatic cancer
•Adhesive capsulitis
FACTORS OTHER THAN INFLAMMATION THAT INCREASE •Anemia
THE ESR •Female sex
•Pregnancy
•Hypercholesterolemia
FACTORS THAT DECREASE THE ESR •Polycythemia
•Sickle cell disease
•Spherocytosis
•Microcytosis (thallasemia, iron deficiency anemia)
COMMON MUSCULOSKELETAL •Polyarthritis
MANIFESTATIONS OF ACROMEGALY •Carpal tunnel syndrome
•Degenerative back pain
•Myopathy
•DJD
COMMON MUSCULOSKELETAL •Fibromyalgia
MANIFESTATIONS OF HYPERTHYROIDISM •Adhesive capsulitis
•Back pain (osteopenia)
•Myopathy (esp proximal)
COMMON MUSCULOSKELETAL •Fibromyalgia
MANIFESTATIONS OF HYPOTHYROIDISM •Polyarthralgia/polyarthritis
•Carpal tunnel syndrome
•Adhesive capsulitis
•Myopathy (esp proximal)
•Delayed relaxation phase of DTRs

4
Part III Handouts v.1.0 Page 127 of 142
PART III Miscellaneous Factoids! CCCLA National Board Review

COMMON MUSCULOSKELETAL •Weakness


MANIFESTATIONS OF ADDISONʼS DISEASE •Fibromyalgia
•Arthralgias
•Joint effusions
•Muscle wasting
COMMON MUSCULOSKELETAL •Painful tibias syndrome
MANIFESTATIONS OF CUSHINGʼS SYNDROME •Joint effusions
•Back pain (osteopenia)
•Osteonecrosis
•Myopathy (esp proximal)
COMMON MUSCULOSKELETAL •Pseudogout
MANIFESTATIONS OF HYPERPARATHYROIDISM •Polyarthralgia/polyarthritis
•Brown tumors
•Back pain (osteopenia)
•Fracture
COMMON MUSCULOSKELETAL •Cheiroarthropathy
MANIFESTATIONS OF DIABETES MELLITUS •Dupuytrenʼs contracture
•Trigger finger
•Adhesive capsulitis
•DISH
•Neuropathic joints
Degenerative spondylolisthesis is more
common in DM
MOST COMMON CAUSES OF HYPERURICEMIA •Uric acid overproduction
Gout (10% of cases)
Overweight
Excessive alcohol use
Psoriasis
Multiple myeloma
•Uric acid underexcretion
Gout (90% of cases)
Renal insufficiency
Excessive alcohol use
Hypertension
Hypothyroidism
Hyperparathyroidism
CONDITIONS ASSOCIATED WITH CPPD •Aging
•OA
•DM
•Hyperparathyroidism
•Hypothyroidism
•Hemochromatosis
CAUSES OF MIGRATORY ARTHRITIS •Hepatitis B or C
•Rheumatic fever
•SLE
•Lyme disease
•Gonoccocal arthritis
•Endocarditis

5
Part III Handouts v.1.0 Page 128 of 142
PART III Miscellaneous Factoids! CCCLA National Board Review

WHEN SHOULD YOU SUSPECT THAT CERVICAL/ARM PAIN IS NON-


MECHANICAL and/or SERIOUS?

When the patient has…

1. Gradually increasing pain. Cervical disc lesions can do this, but only over a short
period of time.
2. Expanding pain area.
3. Bilateral arm pain.
4. Radicular arm pain in a patient younger than 35. Cervical disc lesions are uncommon
in this age group.
5. Arm pain lasting longer than 6 months.
6. Limitation in ALL cervical ranges of motion.
7. Involuntary muscle spasm in passive ROM.
8. Resisted movement of neck is painful AND weak.
9. Lateral flexion away from the painful side is the only painful movement. Worry about
costoscapuloclavicular lesions or apical lung tumor.
10.Scapular elevation is weak and limited. Worry about the brainstem.
11.Hornerʼs syndrome is present (ipsilateral miosis, ptosis, anhydrosis). Worry about
lesion of the cervical sympathetic ganglia from a tumor in the thorax of apical lung
tumor.
12.Hoarseness, especially if painless.
13.T1 palsy. Leads to atrophy and weakness of the intrinsic muscles of the hand. Often
is the first sign of ALS.
14.Paralysis.
15.Three nerve roots are involved. Disc lesions occasionally affect 2 nerve roots, but 3-
root involvement almost always is due to a more serious cause.
16.Muscle weakness in the absence of pain.
17.Dermatomal pain or paresthesia initially present only distally, but then moves
proximally.
18.Babinski sign.

6
Part III Handouts v.1.0 Page 129 of 142
PART III Miscellaneous Factoids! CCCLA National Board Review

WHEN SHOULD YOU SUSPECT THAT THORACIC SPINE PAIN IS NON-


MECHANICAL and/or SERIOUS?

When the patient has…

1. Progressively increasing pain.


2. Expanding area of pain.
3. Continuous pain.
4. Pain not affected by movement, activity, or posture.
5. First symptoms in the thoracic area in a patient older than 50.
6. Central thoracic spine pain radiating bilaterally toward the front. Think of tumor,
extradural hematoma, or, sometimes, herpes zoster (more likely unilateral).
7. Gross limitation of bilateral lateral flexion and rotation. Worry about AS.
8. Gross limitation of extension. Think infection, tumor, AS, DISH.
9. Limitation of lateral flexion away from side of pain as the only restricted motion. Think
lung or abdominal neoplasm or thoracic neuroma.
10.Flexion with a rigid thoracic section. Think AS, fracture, tumor, osteomyelitis.
11.Altered neurological signs in the legs.
12.Signs of multiple root involvement.
13.Involuntary muscle spasm with passive movement.
14.Local paravertebral mass.
15.Gibbus deformity.

7
Part III Handouts v.1.0 Page 130 of 142
PART III Miscellaneous Factoids! CCCLA National Board Review

WHEN SHOULD YOU SUSPECT THAT LOW BACK/LEG PAIN IS NON-


MECHANICAL and/or SERIOUS?

When the patient has…

1. Pain is located in the thoracolumbar /upper lumbar region. Only 5% of mechanical


back pain occurs in the region.
2. Gradually increasing pain.
3. Expanding area of pain.
4. Continuous pain.
5. Sciatica that lasts too long. Radicular pain that has not improved in 9 months is
virtually always due to a serious cause.
6. Bilateral sciatica.
7. Increasing back pain after surgery.
8. Acute low back pain in a geriatric patient.
9. Gross and bilateral limitation of lateral flexion of the lumbar spine. Think of AS,
neoplasm, Pagetʼs, osteomyelitis, or fracture.
10.The only gross restriction in ROM is lateral flexion away from side of pain. Be highly
suspicious of abdominal neoplasm, especially colon or kidney, or a lumbar neuroma.
11.Flexion with a rigid lumbar spine. Think of the seronegative spondyloarthropathies.
12.The degree of weakness is greater than the degree of pain.
13.Involvement of multiple roots.
14.Weakness of the L1 and L2 roots.
15.Saddle paresthesia/anesthesia and incontinence.
16.Presence of the sign of the buttock. While this could be due to a severe disc lesion, it
can also signify sacral, iliac, or femoral malignancy, septic arthritis of the SI joint, or a
rectal abscess.

8
Part III Handouts v.1.0 Page 131 of 142
PART III Miscellaneous Factoids! CCCLA National Board Review

DDX: SI joint sclerosis

ANKYLOSING SPONDYLITIS:
! Male predominance
! Typically bilateral
! Blurring of SI joint
! Loss of subchondral cortex
! Joint irregularly widened
! Sclerosis on either side of joint but more on the ilium side
! Ankylosis of joint

PSORIATIC ARTHRITIS:
! Possibly unilateral
! Erosion of joint margins
! Sclerosis
! Possible ankylosis
! Patient with psoriasis

REACTIVE ARTHRITIS:
! Asymmetrical involvement of SI joints
! Male predominance
! May have ankylosis of joint
! Irregularity of joint margin
! Loss of cortical outline
! Joint sclerosis

OSTEITIS CONDENSANS ILII:


! Occurs in females of childbearing age
! Dense sclerosis along SI joint on ilium side
! Usually following pregnancy
! Bilaterally symmetrical
! Joint space normal

PAGETʼS (OSTEITIS DEFORMANS)


! Male predominance
! Increased trabecular pattern in ilium and sacrum (mosaic)
! Sclerotic bone with osteopenia
! Increase in bone size
! Irregular joint margins

9
Part III Handouts v.1.0 Page 132 of 142
PART III Miscellaneous Factoids! CCCLA National Board Review

CLINICAL DDX OF NEUROMOTOR SYNDROMES

FINDING LMN LESION UMN LESION EXTRA- CEREBELLAR


PYRAMIDAL

ATROPHY Yes Minimal No No

TONE Decreased Increased Increased Possibly


(spasticity) (rigidity) decreased

WEAKNESS Yes (individual Yes (groups of No No


muscles) muscles)

FASCICULATE Yes No No No

GAIT Foot drop and Spastic Festinant, Broad based,


steppage, (circumduction, shuffling ataxic
waddling, scirrors)
quadriceps,
trendelenburg

10
Part III Handouts v.1.0 Page 133 of 142
PART III Miscellaneous Factoids! CCCLA National Board Review

FAST-TRACK NEURO DDX

PROBLEM: ABNORMAL BLADDER CONTROL

Question 1: Are there also bowel control problems?

If NO: Neuro exam is normal = urologic problem (do UA, BUN, creatinine, PSA)
! Neuro exam is abnormal, skip to question 3.

If YES:
Question 2: Are there mental SXs?

If YES = dementia if no motor signs; = brain tumor, stroke, if motor signs.

If NO mental SX:
Question 3a-3c:
! a. Bowel and bladder Sx ONLY? = diabetic autonomic neuropathy
! b. Also perineal paresthesia/numbness? = cauda equina syndrome
! c. Also UMN signs? = spinal cord lesion or brain lesion

PROBLEM: DISTAL SYMMETRICAL WEAKNESS

Question 1: Are the deep tendon reflexes present?

If YES:
! Question 1a. Myotonia is present? = myotonic dystrophy
! Question 1b. Fasciculations are present? = LMN disorder

If NO:
Question 2: Is sensory impairment present?

If YES: Diabetic, alcoholic, B12, or heavy metal neuropathy


If NO: Guillian-Barre (ascending weakness), SLE, tick paralysis

PROBLEM: PROXIMAL SYMMETRICAL WEAKNESS

Question 1: Is swallowing affected?

If YES: Polymyositis (normal reflexes, elevated CPK)

If NO:
Question 2: Are deep tendon reflexes normal?

11
Part III Handouts v.1.0 Page 134 of 142
PART III Miscellaneous Factoids! CCCLA National Board Review

If YES: Limb girdle dystrophy


If NO: Myasthenia gravis, or congenital myopathies, or glycogen storage diseases

PROBLEM: DISTAL AND PROXIMAL SYMMETRICAL WEAKNESS

Question 1: is there a dermatomal sensory abnormality?

If NO: Guillian-Barre syndrome (advanced stage)

If YES:
Question 2: is there involvement of sphincters?

If NO: Radiculopathy
If YES: Paraplegia (legs only) or quadriplegia (arms and legs)

PROBLEM: ASYMMETRICAL WEAKNESS

Question 1: What is the timing of the onset?

INSIDIOUS: CNS mass or degenerative disease (spastic tone and hyperreflexia)

SUDDEN:
Question 2: What is the extent of involvement?

-A group of muscles:
! Innervated by 1 nerve = mononeuropathy; compressive or vascular
! Innervated by a few nerves = plexopathy or polyneuropathy
-One extremity: CNS or multiple root lesions
-Hemiplegia (arm and leg on same side): CNS vascular lesion

PROBLEM: HEARING LOSS

Question 1: Is the hearing loss unilateral or bilateral?


Question 2: What was the timing of the onset?

UNILATERAL:
! ACUTE: infarct or bacterial infection
! CHRONIC PROGRESSIVE: suspect tumor (CT or MRI)

12
Part III Handouts v.1.0 Page 135 of 142
PART III Miscellaneous Factoids! CCCLA National Board Review

BILATERAL:
! ACUTE: viral infection (vestibular neuronitis, labyrinthitis)
! SUBACUTE, INTERMITTENT: Meniereʼs disease (with vertigo and tinnitus)
! CHRONIC PROGRESSIVE: refer for hearing aids

PROBLEM: DIZZINESS

Question 1: What does patient mean by dizziness?

LIGHTHEADEDNESS: Orthostatic hypotension


! ! ! Multiple sensory defects
! ! ! Cervical spine disease

DYSEQUILIBRIUM:!Cerebellar disease
! ! ! Spinal cord disease
! ! ! Basal ganglion disorders

VERTIGO:!
! Question 2: Is vertigo affected by head motion?

If YES:
! Benign positional vertigo (vertigo ONLY with head motion)
! Meniereʼs disease (with hearing loss and tinnitus)
! CVA, TIA, MS (with other brain stem signs)

IF NO:
! Drugs, alcohol, toxins
! Vestibulopathy
! Psychogenic

PROBLEM: AREFLEXIA

Question 1: Is it symmetrical?

If SYMMETRICAL:
Question 2: Is it generalized or segmental?

! Generalized = Normal finding


! ! ! Hypothyroidism (other signs)
! ! ! Adieʼs syndrome (tonic pupils)
! Segmental:
! ! Fasciculations?

13
Part III Handouts v.1.0 Page 136 of 142
PART III Miscellaneous Factoids! CCCLA National Board Review

! ! ! If YES = Anterior horn cell disorder


! ! ! ! LMN disease
! ! ! ! Syringomyelia
! ! ! If NO = myelopathy (will have dermatomal sensory deficit)

If ASYMMETRICAL:
Question 3: What is the distribution?
! One segment = Mononeuropathy
! ! ! Radiculopathy
! Several segments = Plexopathy
! One or two (ipsilateral) extremities = CNS lesion

PROBLEM: SYNCOPE

Question 1: Triggered by specific situation?

If YES:
! Question 2: History of heart disease?
! ! If NO: vasovagal syncope (will have bradycardia during event)
! ! If YES: MI (chest pain, abnormal EKG)
! ! ! Arrhythmia (no chest pain, abnormal EKG)

If NO:
! Other neurologic symptoms? = TIA or other CNS lesion
! Occurs when standing up?
! ! Associated with hypotension = orthostatic hypotension
! ! Not associated with hypotension = aortic stenosis; anemia
! History of heart disease?
! ! MI (chest pain, abnormal EKG)
! ! Pulmonary embolus (chest pain, normal EKG)
! ! Arrhythmia (no chest pain, abnormal EKG)
! ! Using hypertensive medication? = Hypotension (overmedicated)

14
Part III Handouts v.1.0 Page 137 of 142
PART III Miscellaneous Factoids! CCCLA National Board Review

INTERPRETATION OF ABNORMAL RESISTED SHOULDER ROM (most commons)

RROM PAIN PAIN & WEAK WEAKNESS

ABDUCTION Supraspinatus Partial rupture Complete rupture


tendinopathy; deltoid supraspinatus supraspinatus; C5,
lesion subscapular, or axillary
nerve lesion

ADDUCTION Pectoralis major, teres M C7 nerve root lesion


and m, latissimus dorsi
lesion; biceps lesion; AC
joint lesion

INT. ROT. Subscapularis Partial rupture C6 nerve root lesion


tendinopathy subscapularis

EXT. ROT. Infraspinatus tendinopathy Partial rupture Complete rupture


infraspinatus infraspinatus; C5,
suprascapular nerve
lesion

FLEXION Biceps, anterior deltoid Biceps rupture; C5, C6


lesion nerve root lesion

EXTENSION Triceps lesion C7 nerve root lesion

15
Part III Handouts v.1.0 Page 138 of 142
PART III Miscellaneous Factoids! CCCLA National Board Review

Leukemias:

ALL = acute lymphocytic leukemia


AML = acute myelocytic leukemia
CLL = chronic lymphocytic leukemia
CML = chronic myelocytic leukemia

FEATURE ALL AML CLL CML

Peak age of onset Childhood Any age Middle to old age Young adulthood

WBC count H 50%, L-N 50% H 60%, L-N 40% H in 98%, L-N 2% High in 100%

WBC differential Lymphoblasts Myeloblasts Many small Entire myeloid


elevated elevated lymphocytes series high

Anemia In > 90%; severe In > 90%; severe In 50%; mild In 80%; mild

Platelets Low in > 80% Low in > 90% Low in 25% H 60%, L 10%

Lymphadenopathy Yes Occasionally Yes Rare

Splenomegaly 60% 50% Yes, moderate Yes, severe

Other CNS involvement Auer rods seen in Possible WBC alkaline


common myeloblasts hemolytic anemia phosphate low;
and hypogamma- Philadelphia
globulinemia chromosome + in
> 90%

Signs/symptoms Infection, easy Infection, easy Fatigue, anorexia, Fatigue, anorexia,


bruising, bleeding, bruising, bleeding, weight loss, weight loss,
pallor, fatigue, pallor, fatigue, dyspnea on dyspnea on
fever, malaise, fever, malaise, exertion, exertion,
weight loss, weight loss, abdominal abdominal
tachycardia, tachycardia, fullness, pallor, fullness, pallor,
meningitis meningitis rash, pneumonia rash

Prognosis Cure is a realistic Cure is a realistic 3-20 yr survival 3-20 yr survival


goal goal time, depending time, depending
on severity on severity

16
Part III Handouts v.1.0 Page 139 of 142
PART III Miscellaneous Factoids! CCCLA National Board Review

Lymphomas:

HODGKIN NON-HODGKIN

Localized to specific group of lymph Usually disseminated among more than


nodes one nodal group

Tends to spread in an orderly, Spreads noncontiguously


contiguous fashion

Does not usually affect Waldeyerʼs ring* Commonly affects the mesenteric nodes
and the mesenteric nodes and may affect Waldeyerʼs ring*

Infrequently involves extranodal sites Frequently involves extranodal sites

Usually diagnosed at an early stage Usually diagnosed at an advanced stage

In children, usually displays a favorable In children, usually is high grade


histologic classification

Painless cervical adenopathy; pain with Asymptomatic peripheral adenopathy;


alcohol consumption; intense pruritis weakness, fever, night sweats, weight loss,
may occur; Pel-Ebstein** fever, night hepatosplenomegaly, edema; anemia;
sweats, weight loss; splenomegaly;
possbile ivory vertebra, jaundice, and
edema

Cure in 75% of cases Prognosis dependent on numerous


variables; 5-yr survival ranges from 25-75%

*The ring consists of (from superior to inferior):

■ Pharyngeal tonsil (also known as 'adenoids')

■ Tubal tonsils

■ Palatine tonsils (commonly called "the tonsils" in the vernacular, less commonly termed "faucial
tonsils")

■ Lingal tonsils

**a few days of high fever alternating with a few days or weeks of normal or low temperature

17
Part III Handouts v.1.0 Page 140 of 142
PART III Miscellaneous Factoids! CCCLA National Board Review

NEURO-MOST COMMONS

MOST COMMON… [any numbered lists are in decreasing order of frequency]


 S/S of increased intracranial pressure: 1. MORNING HEADACHE, 2. papilledema, 3. projectile
vomiting, 4. bradycardia, 5. hypertension.
 S/S of neurofibromatosis: 1. café-au-lait spots, 2. axillary freckling, 3. pigmented neurofibromas,
4. brain tumors (meningiomas, acoustic neuromas, optic gliomas), 5. pheochromocytoma.
 S/S of syringomyelia: Hornerʼs syndrome, loss of crossed lateral spinothalamic tracts in a cape-
like distribution, atrophied intrinsic muscles of hand
 S/S of meningitis: 1. FEVER, 2. headache, 3. nuchal rigidity (most specific but not most
common), 4. altered sensorium.
 cause of viral meningitis: coxsackievirus.
 cause of bacterial meningitis: S. pneumococcus.
 complications of meningitis: 1. TEMPORARY PARALYSIS, 2. permanent deafness, 3. mental
retardation, 4. hydrocephalus, 5. epilepsy.
 cause of epidural hematoma: fracture of temporoparietal bone causing tearing of middle
meningeal artery.
 S/S of epidural hematoma: immediate loss of consciousness followed by a lucid interval, followed
by death in 5-6 hours.
 cause of subdural hematoma: blunt trauma to skull causing tearing of bridging veins between
dura and arachnoid.
 S/S of subdural hematoma: fluctuating loss of consciousness.
 S/S of an atherosclerotic MCA stroke: 1. usually preceded by TIAs in the same distribution as
stroke, 2. expressive aphasia (dominant hemisphere), 3. contralateral hemiparesis, 4. sensory
changes in upper extremity.
 S/S of atherosclerotic VB stroke: 1. vertigo, 2. ataxia, 3. ipsilateral sensory changes on the face,
4. contralateral motor and sensory changes in the trunk and limbs.
 cause of an intracerebral bleed: hypertension (lenticulostriate arteries).
 sites of intracerebral bleeds: 1. PUTAMEN, 2. thalamus, 3. pons, 4. cerebellum.
 S/S of subarachnoid hemorrhage: 1. SUDDEN ONSET OF VERY SEVERE OCCIPITAL
HEADACHE, 2. loss of consciousness, 3. neurologic deficits.
 risk factors for subarachnoid hemorrhage: 1. hypertension, 2. polycystic kidney disease, 3.
coarctation of the aorta.
 causes of lacunar infarcts: 1. HYPERTENSION, 2. diabetes mellitus.
 S/S of lacunar strokes: pure motor or pure sensory strokes.
 demyelinating disease in CNS: MS.
 S/S of MS: 1. paresthesias, 2. scanning speech, 3. intention tremor, 4. nystagmus, 5. optic
neuritis, 6. ataxia, 7. internuclear opthalmoplegia.
 causes of dementia: 1. Alzheimerʼs, 2. multi-infarct dementia, 3. B12 deficiency, 4. acute
intermittent porphyria, 5. primary hypothyroidism.
 S/S of Parkinsonʼs: 1. COGWHEEL RIGIDITY, 2. BRADYKINESIA (of voluntary activity), 3.
resting tremor, 4. stooped posture, 5. festinating gait, 6. expressionless face.
 S/S of Huntingtonʼs disease: 1. late onset of chorea, 2. extrapyramidal signs, 3. dementia.
 tumor associated with tinnitus and deafness: acoustic neuroma.
 causes of distal sensorimotor peripheral neuropathy: 1. DM, 2. alcohol, 3. lead toxicity, 4.
vincristine, 5. isoniazid, 6. B1 or B12 deficiency.
 cause of acute peripheral neuropathy: Guillian-Barre syndrome.
 cause of Bellʼs palsy: 1. HSV-1, 2. Lyme disease.
 causes of sudden monocular blindness: 1. OPTIC NEURITIS, 2. central retinal artery occlusion,
3. amaurosis fugax, 4. retinal detachment.
 cause of sensorineural hearing loss: presbycusis.

18
Part III Handouts v.1.0 Page 141 of 142
PART III Miscellaneous Factoids! CCCLA National Board Review

NEOPLASIA MOST COMMONS

Most common…
-B9 soft tissue tumor = lipoma
-CA derived from bone marrow = leukemia
-CA derived from lymph nodes = malignant non-Hodgkins lymphoma
-location in bone for met = vertebral bodies
-organ metastasized to = lymph nodes
-malignancy to met to lymph nodes = breast CA
-malignancy to met to lungs = breast CA
-malignancy to met to brain = lung CA
-malignancy to met to bone = breast CA
-primary site for osteoblastic met = prostate CA
-enzyme elevated in osteoblastic met = alk phos
-malignancies that produce purely osteolytic met = lung, kidney
-CA associated with fever not due to infection = Hodgkin’s disease
-CAs associated with secretion of hCG = choriocarcinoma, testicular CA
-tumors associated with release of serotonin = carcinoid tumors in terminal ileum with met to
liver
-cancers associated with secretion of AFP = 1. hepatocellular CA, 2. ovarian CA, 3. testicular
CA
-tumor markers ordered to R/O testicular CA = AFP, hCG
-tumor markers ordered to R/O surface-derived ovarian CA = CA 125
-tumor markers ordered to R/O small-cell CA of lung = CEA
-tumor markers ordered to R/O prostate CA = PSA
-tumor markers ordered to R/O multiple myeloma = Bence-Jones proteins in urine
(however, only 40% or Pts have this; serum protein electrophoresis is better)
-tumor markers to R/O breast CA = CEA & CA 15-3
-tumor markers to R/O colon CA = CEA
-tumor markers to R/O pancreatic CA = CA 19-9 & CEA
-cancers (in decreasing order of incidence) in men = prostate, lung, colorectal
-cancers (in decreasing order of incidence) in women = breast, lung, colorectal
-cancer mortalities (in decreasing order of incidence) in men = lung, prostate, colorectal
-cancer mortalities (in decreasing order of incidence) in women = lung, breast, colorectal
-inherited syndrome associated with acoustic neuromas = neurofibromatosis
-most common sources of brain metastasis: 1. breast, 2. lung, 3. melanoma

19
Part III Handouts v.1.0 Page 142 of 142

Você também pode gostar